ORTHOPEDIC MCQS WITH ANSWERS ONLINE TRAUMA 06

ORTHOPEDIC MCQS WITH ANSWERS ONLINE TRAUMA 06

1.      A 36-year-old woman sustained a tarsometatarsal joint fracture-dislocation in a motor vehicle accident.  The patient is treated with open reduction and internal fixation.  What is the most common complication?

 

1-         Posttraumatic arthritis

2-         Infection

3-         Fixation failure

4-         Malunion

5-         Nonunion

 

PREFERRED RESPONSE: 1

 

DISCUSSION: The most common complication associated with tarsometatarsal joint injury is posttraumatic arthritis.  In one series, symptomatic arthritis developed in 25% of the patients and half of those went on to fusion.  In another series, 26% had painful arthritis.  Initial treatment should consist of shoe modification, inserts, and anti-inflammatory drugs.  Fusion is reserved for failure of nonsurgical management.  Hardware failure may occur, but it is clinically unimportant.

 

REFERENCES: Kuo RS, Tejwani NC, DiGiovanni CW, et al: Outcome after open reduction and internal fixation of Lisfranc joint injuries.  J Bone Joint Surg Am 2000;82:1609-1618.

Arntz CT, Veith RG, Hansen ST Jr: Fractures and fracture-dislocations of the tarsometatarsal joint.  J Bone Joint Surg Am 1988;70:173-181.

Thompson MC, Mormino MA: Injury to the tarsometatarsal joint complex.  J Am Acad Orthop Surg 2003;11:260-267.

 

2.      What is the most appropriate indication for replantation in an otherwise healthy
35-year-old man?

 

1-         Isolated transverse amputation of the thumb through the middle of the nail bed

2-         Isolated transverse amputation of the index finger through the proximal phalanx

3-         Isolated transverse amputation of the ring finger through the proximal phalanx

4-         Isolated transverse amputation of the hand at the level of the wrist

5-         Forearm amputation with a 10-hour warm ischemia time

 

PREFERRED RESPONSE: 4

 

DISCUSSION: Vascular anastamoses are exceedingly difficult with amputations distal to the nail fold as the digital vessels bifurcate or trifurcate at this level, and little functional benefit is gained compared to other means of soft-tissue coverage.  Single digit amputations, other than
the thumb, are a relative contraindication for replantation.  Replantations at the level of the proximal phalanx lead to poor motion of the proximal interphalangeal joint.  In a healthy
active adult, an amputation through the wrist is an appropriate situation to proceed with a replantation.  A transverse forearm amputation is a good indication with a warm ischemia time
of less than 6 hours.

 

REFERENCES: Urbaniak JR: Replantation, in Green DP, Hotchkiss RN (eds): Operative Hand Surgery, ed 3.  New York, NY, Churchill Livingstone, 1993, p 1085.

Boulas HJ:  Amputations of the fingers and hand: Indications for replantation.  J Am Acad Orthop Surg 1998;6:100-105.

 

3.      Figure 1 shows the radiograph of an 11-year-old boy who stubbed his great toe while playing soccer barefoot.  He is able to walk home despite a small amount of bleeding at the nail fold.  Management should consist of

 

1-         a hard-soled shoe for 4 to 6 weeks.

2-         closed reduction and percutaneous pinning.

3-         burning a hole in the nail for relief of the impending hematoma.

4-         digital block and irrigation and debridement of the wound.

5-         repair of the extensor hallucis longus tendon injury and use of a hard-soled shoe.

 

PREFERRED RESPONSE: 4

 

DISCUSSION: Great toe fractures through the physis should be considered open fractures if there is bleeding at the nail fold.  Treatment should include irrigation and debridement and appropriate antibiotics.  Immobilization with a cast is usually sufficient for treatment of
the fracture.

 

REFERENCES: Noonan KJ, Saltzman CL, Dietz FR: Open physeal fractures of the distal phalanx of the great toe: A case report.  J Bone Joint Surg Am 1994;76:122-125.

Banks AS, Cain TD, Ruch JA: Physeal fractures of the distal phalanx of the hallux.  J Am Podiatr Med Assoc 1988;78:310-313.

 

4.      Figure 2a shows the radiograph of a 48-year-old man who was involved in a motorcycle accident.  A CT scan is shown in Figure 2b.  The patient underwent pelvic angiography for persistent hypotension despite resuscitation.  What vessel is most likely to be injured?

 

1-         Internal iliac

2-         External iliac

3-         Pudendal

4-         Superior rectal

5-         Superior gluteal

 

PREFERRED RESPONSE: 5

 

DISCUSSION: The pelvic injury is a severe anterior-posterior compression III or Tile C injury.  The vessel most likely injured is the superior gluteal artery, but several arterial bleeding sources are likely.  Vertical shear injuries can also injure this vessel, but it is much less common.  When arterial injury follows a lateral compression injury, it is usually related to injury of a more anterior vessel like the obturator artery or a branch of the external iliac artery.

 

REFERENCES: O’Neill PA, Riina J, Sclafani S, et al: Angiographic findings in pelvic fractures.  Clin Orthop 1996;329:60-67.

Belley G, Gallix BP, Derossis AM, et al: Profound hypotension in blunt trauma associated with superior gluteal artery rupture without pelvic fracture.  J Trauma 1997;43:703-705.

 

5.      A 46-year-old man fell 20 feet and sustained the injury shown in Figure 3.  The injury is closed; however, the soft tissues are swollen and ecchymotic with blisters.  The most appropriate initial management should consist of

 

1-         a long leg cast.

2-         a short leg cast.

3-         immediate open reduction and internal fixation.

4-         a temporizing spanning external fixator.

5-         primary ankle fusion.

 

PREFERRED RESPONSE: 4

 

DISCUSSION: Although this is a fracture of the medial and lateral malleoli, the degree of displacement and comminution of the medial dome indicate that this injury is similar to a pilon fracture.  Initial management should consistent of stabilization to allow for soft-tissue healing.  The use of temporizing spanning external fixation should be the initial step, followed by limited or more extensive open reduction and internal fixation when the soft-tissue status will allow.  Initial placement in either a short or long leg cast does not provide the needed stability and does not allow for care and monitoring of soft tissues.  In addition, maintaining reduction of the talus may be very difficult.  Immediate open reduction and internal fixation through an injured soft-tissue envelope adds the risk of difficulties with incision healing and a higher risk of deep infection.  In the acute setting, a primary ankle fusion through this soft-tissue envelope is
not indicated.

 

REFERENCES: Marsh JL, Bonar S, Nepola JV, et al: Use of an articulated external fixator for fractures of the tibial plafond.  J Bone Joint Surg Am 1995;77:1498-1509.

Wyrsch B, McFerran MA, McAndrew M, et al: Operative treatment of fractures of the tibial plafond: A randomized, prospective study.  J Bone Joint Surg Am 1996;78:1646-1657.

Thordarson DB: Complications after treatment of tibial pilon fractures: Prevention and management strategies.  J Am Acad Orthop Surg 2000;8:253-265.

 

6.      A 13-year-old boy hyperextends his knee while playing basketball and reports a pop that is followed by a rapid effusion.  A lateral radiograph is shown in Figure 4.  Initial management consists of attempted reduction with extension, with no change in position of the fragment.  What is the next most appropriate step in management?

 

1-         A long leg cast in 90° of knee flexion for 6 weeks

2-         Open reduction and internal fixation with a transphyseal 6.5-mm screw and washer

3-         Arthroscopic anterior cruciate ligament reconstruction with hamstring tendons

4-         Arthroscopic debridement and staged anterior cruciate ligament reconstruction when skeletally mature

5-         Open reduction and internal fixation with suture or intra-epiphyseal screw placement

 

PREFERRED RESPONSE: 5

 

DISCUSSION: Avulsion fractures of the tibial spine are rare injuries that result from rapid deceleration or hyperextension of the knee in skeletally immature individuals.  This injury is the equivalent of ruptures of the anterior cruciate ligament in adults.  These fractures are classified as types 1 through 3.  Type 1 is a minimally displaced fracture, type 2 fractures have an intact posterior hinge, and type 3 fractures have complete separation.  The radiograph demonstrates a completely displaced, or type III, tibial spine avulsion.  Surgical reduction is indicated in type 2 fractures that fail to reduce with knee extension and in all type 3 fractures.  Reduction may be arthroscopic or open, with fixation of the bony fragment using a method that maintains physeal integrity and prevents later growth arrest.  Preferred techniques would be with suture or an intra-epiphyseal screw

 

REFERENCES: Wiley JJ, Baxter MP: Tibial spine fractures in children.  Clin Orthop 1990;255:54-60.

Mulhall KJ, Dowdall J, Grannell M, et al: Tibial spine fractures: An analysis of outcome in surgically treated type III injuries.  Injury 1999;30:289-292.

Owens BD, Crane GK, Plante T, et al: Treatment of type III tibial intercondylar eminence fractures in skeletally immature athletes.  Am J Orthop 2003;32:103-105.

Vocke AK, Vocke AR: Cartilaginous avulsion fracture of the tibial spine.  Orthopedics 2002;25:1293-1294.

 

7.      A collegiate golfer sustains a hook of the hamate fracture.  After 12 weeks of splinting and therapy, the hand is still symptomatic.  What is the most appropriate management to allow return to competitive activity?

 

1-         Continued observation

2-         Open reduction and internal fixation of the fracture

3-         Excision of the hook of the hamate

4-         Carpal tunnel release

5-         Guyon’s canal release

 

PREFERRED RESPONSE: 3

 

DISCUSSION: Excision of the fracture fragment typically leads to rapid return to function.  Fixation techniques are difficult to perform because of the size of the bone; hardware prominence is common.  Nerve deficits are not typically noted in this injury.  The motor branch of the ulnar nerve in Guyon’s canal must be protected during the surgical approach.

 

REFERENCES: Kulund DN, McCue FC III, Rockwell DA, et al: Tennis injuries: Prevention and treatment: A review.  Am J Sports Med 1979;7:249-253.

Morgan WJ, Slowman LS: Acute hand and wrist injuries in athletes: Evaluation and management.  J Am Acad Orthop Surg 2001;9:389-400.

 

8.      In an acute closed boutonniere injury, what is the most appropriate splinting technique for the proximal interphalangeal joint?

 

1-         Static splint in 30° of flexion

2-         Static splint in full extension

3-         Dynamic extension splint

4-         30° extension block splint

5-         Buddy taping to the adjacent finger

 

PREFERRED RESPONSE: 2

 

DISCUSSION: Rupture of the central slip of the extensor mechanism and a varying degree of lateral band volar migration are the pathologic entities in an acute boutonniere injury.  Splinting the proximal interphalangeal joint in full extension allows reapproximation of the central slip to the base of the middle phalanx.  Distal interphalangeal joint flexion is permitted to allow movement of the lateral bands distally and dorsally, preventing contracture.

 

REFERENCES: Newport ML: Extensor tendon injuries in the hand.  J Am Acad Orthop Surg 1997;5:59-66.

Lovet WL, McCalla MA: Management and rehabilitation of extensor tendon injuries.  Orthop Clin North Am 1983;14:811-826.

 

9.      A 20-year-old man sustained a closed tibial fracture and is treated with a
reamed intramedullary nail.  What is the most common complication associated
with this treatment?

 

1-         Nonunion

2-         Malunion

3-         Infection

4-         Knee pain

5-         Compartment syndrome

 

PREFERRED RESPONSE: 4

 

DISCUSSION: The most common complication is anterior knee pain (57%).  The knee pain is activity related (92%) and exacerbated by kneeling (83%).   Although knee pain is the most common complication, most patients rate it as mild to moderate and only 10% are unable to return to previous employment.  Some authors report less knee pain with a peritendinous approach when compared to a tendon-splitting approach.  In one study, nail removal resolved pain in 27%, improved it in 70%, and made it worse in 3%.  The incidence of the other complications was: infection 0% to 3%, nonunion 0% to 6%, and malunion 2% to 13%.  Compartment syndrome is rare after nailing. 

 

REFERENCES: Court-Brown CM: Reamed intramedullary tibial nailing: An overview and analysis of 1106 cases.  J Orthop Trauma 2004;18:96-101.

McQueen MM, Gaston P, Court-Brown CM: Acute compartment syndrome: Who is at risk? 
J Bone Joint Surg Br 2000;82:200-203.

Keating JF, Orfaly R, O’Brien PJ: Knee pain after tibial nailing.  J Orthop Trauma
1997;11:10-13.

 

10.    An 8-year-old boy sustained an isolated distal radial fracture that was reduced and immobilized with 10° of residual dorsal tilt.  What is the next step in management?

 

1-         Percutaneous pinning

2-         Open reduction and pin fixation

3-         Follow-up in 6 weeks for conversion to a splint

4-         A short arm cast and follow-up in 4 weeks

5-         A long arm cast and follow-up in 1 week

 

PREFERRED RESPONSE: 5

 

DISCUSSION: Distal radial fractures in children are common, and a large amount of displacement is acceptable.  In general, 20° of dorsal displacement and complete bayonet apposition in girls to age 12 years and in boys to age 14 years can be expected to remodel with an excellent outcome.  The potential for increased fracture displacement and subsequent malunion may exist in up to one third of patients with displaced fractures with less than anatomic reduction.  Therefore, early follow-up is recommended and remanipulation is indicated should loss in reduction occur.  Consideration for percutaneous pinning of isolated distal radial fracture is reasonable in patients with little growth remaining.  In these patients, higher rates of redisplacement exist with little chance for remodeling.

 

REFERENCES: Gibbons CL, Woods DA, Pailthorpe C, et al: The management of isolated distal radius fractures in children.  J Pediatr Orthop 1994;14:207-210.

McLauchlan GJ, Cowan B, Annan IH, et al: Management of completely displaced metaphyseal fractures of the distal radius in children.  J Bone Joint Surg Br 2002;84:413-417.

Proctor MT, Moore DJ, Patterson JH: Redisplacement after manipulation of distal radial fractures in children.  J Bone Joint Surg Br 1993;75:453-454.

Roy DR: Completely displaced distal radius fractures with intact ulnas in children.  Orthopedics 1989;12:1089-1092.

 

11.     A 46-year-old man has incomplete paraplegia after being involved in a motor vehicle accident.  The CT scan shown in Figure 5 reveals marked canal compromise.  What is the most appropriate management to improve neurologic status?

 

1-         Postural reduction and application of a hyperextension cast

2-         Posterior laminectomy alone

3-         Laminectomy and posterior stabilization

4-         Bed rest for 6 weeks

5-         Anterior L2 corpectomy and iliac crest strut grafting, with or without posterior instrumentation and fusion from L1 to L3

 

PREFERRED RESPONSE: 5

 

DISCUSSION: According to a study by the Scoliosis Research Society, the use of anterior decompression is most predictable for improving neurologic status.  This is particularly true of bowel and bladder functional loss.  Laminectomy is contraindicated because it further destabilizes the spine.  Posterior instrumentation and indirect reduction through distraction and ligamentotaxis only incompletely decompress the compromised canal and are successful only if performed within 48 hours of injury.  While some improvement may occur with closed management, the amount of recovery is less than that achieved with surgical decompression.  A posterior approach and instrumentation may be added to the anterior decompression based on the characteristics of associated injuries to the posterior element.

 

REFERENCES: Gertzbein SD: Scoliosis Research Society multicenter spine fracture study.  Spine 1992;17:528-540.

Garfin SR, Vaccaro AR (eds): Orthopaedic Knowledge Update: Spine.  Rosemont, IL, American Academy of Orthopaedic Surgeons, 1997, pp 197-215.

Cammisa FP Jr, Eismont FJ, Green BA: Dural laceration occurring with burst fractures and associated laminar fractures. J Bone Joint Surg Am 1989;71:1044-1052.

 

12.     What is the most likely complication following treatment of the humeral shaft fracture shown in Figure 6?

 

1-         Nonunion

2-         Shoulder pain

3-         Infection

4-         Elbow injury

5-         Radial nerve injury

 

PREFERRED RESPONSE: 2

 

DISCUSSION: The humerus was treated with an intramedullary nail.  Findings from two prospective randomized studies of intramedullary nailing or compression plating of acute humeral fractures have shown approximately a 30% incidence of shoulder pain with antegrade humeral nailing.  This is the most common complication in both of these series.  Nonunions are present in approximately 5% to 10% of humeral fractures treated with an intramedullary nail.  Infection has an incidence of approximately 1%.  Elbow injury is unlikely unless the nail is excessively long.  Rarely, injury to the radial nerve is possible if it is trapped in the intramedullary canal.

 

REFERENCES: Chapman JR, Henley MB, Agel J, et al: Randomized prospective study of humeral shaft fracture fixation: Intramedullary nails versus plates.  J Orthop Trauma 2000;14:162-166.

McCormack RG, Brien D, Buckley RE, et al: Fixation of fractures of the shaft of the humerus by dynamic compression plate or intramedullary nail: A prospective, randomised trial.  J Bone Joint Surg Br 2000;82:336-339.

 

13.     A 4-year-old girl sustains an isolated spiral femoral fracture after falling from her tricycle.  Management should consist of

 

1-         external fixation.

2-         plate fixation.

3-         skeletal traction for 5 weeks.

4-         immediate spica cast immobilization.

5-         flexible nailing with titanium nails.

 

PREFERRED RESPONSE: 4

 

DISCUSSION: Immediate spica casting is ideal for younger children with uncomplicated femoral fractures that are the result of relatively low-energy injury.  Surgical stabilization of pediatric femoral fractures is most commonly performed in children who are older than age 6 years or in children with other factors associated with their femoral fracture, such as concomitant head injury, open fracture, floating knee, severe comminution, or vascular injury. 

 

REFERENCES: Flynn JM, Skaggs DL, Sponseller PD, et al: The surgical management of pediatric fractures of the lower extremity.  Instr Course Lect 2003;52:647-659.

Sponseller PD: Surgical management of pediatric femoral fractures.  Instr Course Lect 2002;51:361-365.

Wright JG: The treatment of femoral shaft fractures in children: A systematic overview and critical appraisal of the literature.  Can J Surg 2000;43:180-189. 

Levy J, Ward WT: Pediatric femur fractures: An overview of treatment.  Orthopedics 1993;16:183-190.

 

14.     A 16-year-old girl sustained the injury shown in Figure 7a.  CT scans are shown in Figures 7b through 7d.  The results of treatment of this injury have been shown to most correlate with which of the following factors?

 

1-         Surgical approach

2-         Location of the transverse fracture

3-         Timing of surgery

4-         Accuracy of reduction

5-         Use of skeletal traction

 

PREFERRED RESPONSE: 4

 

DISCUSSION: The patient has a very low T-type acetabular fracture; however, the head is not congruent under the dome so surgical reduction is necessary.  The anterior and posterior columns are displaced and will move independent from each other.  The extended iliofemoral is the only approach allowing for visualization and reduction of each column.  A combined anterior and posterior approach may also be used.  The timing of surgery should be within the first 3 weeks of injury to optimize chances of obtaining an accurate reduction because this is an important factor in determining outcome.

 

REFERENCES: Letournel E, Judet R (eds): Fractures of the Acetabulum, ed 2. 
Berlin, Germany, Springer-Verlag, 1991.

Matta JM: Fractures of the acetabulum: Accuracy of reduction and clinical results in patients managed operatively within three weeks after the injury.  J Bone Joint Surg Am
1996;78:1632-1645.

 

15.     An 18-month-old child sustains a crush amputation of the tip of the index finger.  Bone is exposed, but the nail is intact.  Management should consist of

 

1-         dressing changes and healing by secondary intention.

2-         a split-thickness skin graft.

3-         a full-thickness skin graft.

4-         a thenar flap.

5-         a V-Y flap.

 

PREFERRED RESPONSE: 1

 

DISCUSSION: Children have a much greater capacity to heal soft-tissue injuries than adults.  Most crush or avulsion fingertip amputations in children, particularly those younger than age 2 years, can be treated with serial dressing changes, even with bone exposed.

 

REFERENCES: Das SK, Brown HG: Management of lost finger tips in children. 
Hand 1978;10:16-27.

Fassler PR: Fingertip injuries: Evaluation and treatment.  J Am Acad Orthop Surg
1996;4:84-92.

 

16.     An otherwise healthy 25-year-old man sustained a wound with a 1-cm by 1.5-cm soft-tissue loss over the volar aspect of the middle phalanx of his middle finger.  After appropriate debridement and irrigation, the flexor digitorum profundus tendon and neurovascular bundles are visible.  The wound should be treated with a

 

1-         split-thickness skin graft.

2-         thenar flap.

3-         cross-finger flap.

4-         lateral arm flap.

5-         Moberg (volar advancement) flap.

 

PREFERRED RESPONSE: 3

 

DISCUSSION: The wound described indicates loss of soft tissue directly to the level of the tendon, precluding use of skin grafts if excursion of the tendon is desired.  A cross-finger
flap is ideal for small wounds on the volar aspect of digits.  A thenar flap is suitable for tip injuries.  A lateral arm flap will not reach the fingers.  A Moberg flap is limited to distal injuries of the thumb. 

 

REFERENCES: Kappel DA, Burech JG: The cross-finger flap: An established reconstructive procedure.  Hand Clin 1985;1:677-683.

Lister GD: Skin flaps, in Green DP, Hotchkiss RN (eds): Operative Hand Surgery, ed 3.  New York, NY, Churchill Livingstone, 1993, p 1741.

 

17.    An active 49-year-old woman who sustained a diaphyseal fracture of the clavicle 8 months ago now reports persistent shoulder pain with daily activities.  An AP radiograph is shown in Figure 8.  Management should consist of

 

1-         external electrical stimulation.

2-         external ultrasound stimulation.

3-         implanted electrical stimulation.

4-         closed reduction and percutaneous fixation.

5-         open reduction and internal fixation with bone graft.

 

PREFERRED RESPONSE: 5

 

DISCUSSION: The radiograph reveals an atrophic nonunion of the diaphysis of the clavicle.  Electrical or ultrasound stimulation may be an option in diaphyseal nonunions that have
shown some healing response with callus formation, but these techniques are not successful
in an atrophic nonunion.  The preferred technique for achieving union is open reduction and internal fixation with bone graft.  Percutaneous fixation has no role in treatment of nonunions
of the clavicle.

 

REFERENCES: Boyer MI, Axelrod TS: Atrophic nonunion of the clavicle: Treatment by compression plating, lag-screw fixation and bone graft.  J Bone Joint Surg Br 1997;79:301-303.

Simpson NS, Jupiter JB: Clavicular nonunion and malunion: Evaluation and surgical management.  J Am Acad Orthop Surg 1996;4:1-8.

 

18.    Examination of a 25-year-old man who was injured in a motor vehicle accident reveals a fracture-dislocation of C5-6 with a Frankel B spinal cord injury.  He also has a closed right femoral shaft fracture and a grade II open ipsilateral midshaft tibial fracture.  Assessment of his vital signs reveals a pulse rate of 45/min, a blood pressure of 80/45 mm Hg, and respirations of 25/min.  A general surgeon has assessed the abdomen, and a peritoneal lavage is negative.  His clinical presentation is most consistent with what type of shock?

 

1-         Neurogenic

2-         Hemorrhagic

3-         Spinal

4-         Septic

5-         Hypovolemic

 

PREFERRED RESPONSE: 1

 

DISCUSSION: Assessment of the acutely injured patient follows the Advanced Trauma Life Support protocol.  Cervical cord injury is often associated with a disruption in sympathetic outflow.  Absent sympathetic input to the lower extremities leads to vasodilatation, decreased venous return to the heart, and subsequent hypotension.  With hypotension, the physiologic response of tachycardia is not possible because of the unopposed vagal tone.  This results in bradycardia.  Patient positioning, fluid support, pressor agents, and atropine are used to treat neurogenic shock.

 

REFERENCE: Sutton DC, Siveri CP, Cotler JM: Initial evaluation and management of the spinal injured patient, in Cotler JM, Simpson JM, An HS, et al (eds): Surgery of Spinal Trauma.  Philadelphia, PA, Lippincott Williams & Wilkins, 2000, pp 113-126.

 

19.    A 32-year-old woman sustained an injury to her left upper extremity in a motor vehicle accident.  Examination reveals a 2-cm wound in the mid portion of the dorsal surface of the upper arm and deformities at the elbow and forearm; there are no other injuries.  Her vital signs are stable, and she has a base deficit of minus 1 and a lactate level of less
than 2.  Radiographs are shown in Figures 9a and 9b.  In addition to urgent debridement of the humeral shaft fracture, management should include

 

1-         closed management of the medial condyle and humeral shaft fractures and open reduction and internal fixation of the both bones forearm fracture.

2-         closed management of the humeral shaft fracture and open reduction and internal fixation of the medial condyle and the both bones forearm fractures.

3-         open reduction and internal fixation of the humeral shaft, medial condyle, and the both bones forearm fractures.

4-         open reduction and internal fixation of the medial condyle and both bones forearm fractures, and external fixation of the humeral shaft fracture.

5-         delayed stabilization of all fractures after the open wound has healed.

 

PREFERRED RESPONSE: 3

 

DISCUSSION: With a severe injury to the upper extremity, the best opportunity for achieving a good functional result for a floating elbow is immediate debridement of the open fracture, followed by internal fixation of the fractures.  The ability to do this depends on the patient’s physiologic status.  In this patient, the procedure is acceptable because she has normal vital signs and no chest or abdominal injuries, and normal physiologic parameters (base excess and lactate) show adequate peripheral perfusion.  The surgical approaches will be determined by the associated injury patterns and open wounds.  In this patient, the humerus was debrided and stabilized through a posterior approach as was the medial condyle fracture.  The ulna was fixed through an extension of the posterior incision and the radius through a separate dorsal approach.

 

REFERENCES: Solomon HB, Zadnik M, Eglseder WA: A review of outcomes in 18 patients with floating elbow.  J Orthop Trauma 2003;17:563-570.

Pape HC, Hildebrand F, Pertschy S, et al: Changes in the management of femoral shaft fractures in polytrauma patients: From early total care to damage control orthopedic surgery.  J Trauma 2002;53:452-461.

 

20.    A patient sustained the injuries shown in the radiographs and clinical photograph seen in Figures 10a through 10c.  The neurovascular examination is normal.  The first step in emergent management of the extremity injuries should consist of

 

1-         application of a femoral traction pin.

2-         intramedullary nailing of the femur and tibia.

3-         surgical irrigation and debridement.

4-         external fixation of the femoral fracture.

5-         reduction of the femoral head.

 

PREFERRED RESPONSE: 5

 

DISCUSSION: The figures show an open tibial fracture, a femoral shaft fracture, and femoral head dislocation.  The most urgent treatment is reduction of the femoral head, as timing to reduction has been correlated with preventing osteonecrosis.  After reduction of the femoral head, the next priority is wound management, followed by stabilization of the femoral and tibial fractures with either splinting, traction, or external fixation. 

 

REFERENCES: Sahin V, Karakas ES, Aksu S, et al: Traumatic dislocation and fracture-dislocation of the hip: A long-term follow-up study.  J Trauma 2003;54:520-529.

Moed BR, WillsonCarr SE, Watson JT: Results of operative treatment of fractures of the posterior wall of the acetabulum.  J Bone Joint Surg Am 2002;84:752-758.

 

21.     Figure 11 shows the radiograph of a 3-year-old girl who sustained a proximal radius injury.  Appropriate initial management should include

 

1-         open reduction.

2-         closed reduction and transarticular pinning.

3-         closed reduction.

4-         a sling and early range of motion.

5-         radial head excision.

 

PREFERRED RESPONSE: 3

 

DISCUSSION: The patient has a displaced radial neck fracture.  Displaced radial neck fractures with angulation of more than 30° to 45° require reduction.  Methods of attempted closed reduction include wrapping the arm with an Esmarch’s bandage and applying direct pressure over the maximum deformity of the radial head.  More aggressive methods include a Kirschner wire used as a joystick or intramedullary reduction as described by the Metaizeau technique.  Open reduction should be avoided because of complications such as stiffness or osteonecrosis.  Indications for open reduction are irreducible displacement of more than 45° with severe restriction of forearm rotation.

 

REFERENCES: Leung AG, Peterson HA: Fractures of the proximal radial head and neck in children with emphasis on those that involve the articular cartilage.  J Pediatr Orthop
2000;20:7-14.

Radomisli TE, Rosen AL: Controversies regarding radial neck fractures in children.  Clin Orthop 1998;353:30-39.

Skaggs DL, Mirzayan R: The posterior fat pad sign in association with occult fracture of the elbow in children.  J Bone Joint Surg Am 1999;81:1429-1433.

Gonzalez-Herranz P, Alvarez-Romera A, Burgos J, et al: Displaced radial neck fractures in children treated by closed intramedullary pinning (Metaizeau technique).  J Pediatr Orthop 1997;17:325-331.

 

22.    Figures 12a and 12b show the radiographs of a 56-year-old man with diabetes mellitus who has had left foot swelling with no pain for the past several weeks.  He denies any history of trauma.  Examination reveals warmth, moderate swelling, no tenderness, and mild pes planus with standing.  Pulses are palpable, and his sensory examination is grossly intact to light touch.  Standing radiographs are shown in Figures 12c and 12d.  What is the most likely diagnosis?

 

1-         Acute traumatic Lisfranc fracture-dislocation

2-         Acquired pes planus due to rupture of the posterior tibial tendon

3-         Neuropathic arthropathy

4-         Osteomyelitis

5-         Metatarsal stress fracture

 

PREFERRED RESPONSE: 3

 

DISCUSSION: The radiographs show tarsometatarsal joint subluxation without fragmentation.  The clinical history and delay in presentation with the radiographic findings suggest a neuropathic or Charcot arthropathy involving the midfoot area.  Intact sensory examination to light touch is not diagnostic for an intact peripheral neurologic system; monofilament testing is a more accurate office baseline examination for the presence of sensory peripheral neuropathy.  With an acute traumatic Lisfranc fracture-dislocation, a history of a traumatic event is necessary, and radiographic abnormalities are expected, although nonstanding radiographs still may be misleading.  Acquired pes planus due to posterior tibial tendon rupture may have negative nonstanding radiographs.  Standing radiographs may reveal pes planus.  However, intermetatarsal disruption is not expected as seen in a Lisfranc abnormality.  Localized osteomyelitis of the foot without a penetrating injury or cutaneous ulceration is extremely unlikely and does not fit with the clinical picture described.  An isolated metatarsal stress fracture would show osseous irregularity without the instability pattern pictured.

 

REFERENCES: Brodsky JW: The diabetic foot, in Coughlin MJ, Mann RA (eds): Surgery of the Foot and Ankle, ed 7.  St Louis, MO, Mosby, 1999, pp 895-969.

Myerson MS: Diabetic neuroarthropathy, in Myerson MS (ed): Foot and Ankle Disorders.  Philadelphia, PA, WB Saunders, 2000, pp 439-465.

 

23.     A 25-year-old student sustains the injury shown in Figures 13a through 13c after falling off a curb.  Initial management should consist of

 

1-         weight bearing as tolerated in a hard-soled shoe.

2-         weight bearing as tolerated in an ankle lacer.

3-         weight bearing as tolerated in a short leg cast.

4-         non-weight-bearing in a hard-soled shoe.

5-         non-weight-bearing in a short leg cast.

 

PREFERRED RESPONSE: 5

 

DISCUSSION: The radiographs reveal a fracture entering the 4-5 intermetatarsal articulation, consistent with a zone 2 injury.  This classically is also referred to as a Jones fracture.  The history and radiographic findings indicate this is an acute fracture, which guides management.  A zone 1 fracture enters the fifth tarsometatarsal joint, and a zone 3 fracture is a proximal diaphyseal fracture distal to the 4-5 articulation.  Initial management is usually nonsurgical and consists of non-weight-bearing in a short leg cast.  This method has been shown to result in a better healing rate compared to weight bearing as tolerated. 

 

REFERENCES: Rosenberg GA, Sterra JJ: Treatment strategies for acute fractures and nonunions of the proximal fifth metatarsal.  J Am Acad Orthop Surg 2000;8:332-338.

Lawrence SJ, Botte MJ: Jones’ fracture and related fractures of the proximal fifth metatarsal.  Foot Ankle 1993;14:358-365.

 

24.    What structure is most often injured in a volar proximal interphalangeal joint dislocation?

 

1-         Sagittal bands

2-         Central slip

3-         Lumbrical

4-         Juncturae tendinum

5-         Terminal extensor tendon

 

PREFERRED RESPONSE: 2

 

DISCUSSION: Closed ruptures of the central slip of the extensor tendon may occur with volar proximal interphalangeal joint dislocation, forced flexion of the proximal interphalangeal joint, or blunt trauma to the dorsum of the proximal interphalangeal joint.  The other structures are not typically injured in proximal interphalangeal joint dislocations.  Treatment typically requires static splinting of the proximal interphalangeal joint.  In the more common dorsal proximal interphalangeal joint dislocation, the volar plate is injured, and early range of motion may be started after reduction.

 

REFERENCES: Doyle JR: Extensor tendons: Acute injuries, in Green DP, Hotchkiss RN (eds): Operative Hand Surgery, ed 3.  New York, NY, Churchill Livingstone, 1993, p 1925.

Newport ML: Extensor tendon injuries in the hand.  J Am Acad Orthop Surg 1997;5:59-66.

 

25.    What patient factor is predictive of better outcomes for surgical management of a displaced calcaneal fracture compared to nonsurgical management?

 

1-         Young man injured at the work site

2-         Young woman injured during recreational activities

3-         Heavy smoker

4-         Patient older than age 50 years

5-         Patient with bilateral fractures

 

PREFERRED RESPONSE: 2

 

DISCUSSION: A recent randomized trial of surgical versus nonsurgical management of calcaneal fractures showed that patients who were on workers’ compensation did poorly with surgical care.  These patients had less favorable outcomes regardless of their initial management.  Factors such as age, smoking, and vasculopathies compromise skin healing, leading to greater surgical risks.  The best results were obtained in patients who are younger than age 40 years, have unilateral injuries and are injured during noncompensable activities.  Women tend to do better with surgery than men.

 

REFERENCES: Howard JL, Buckley R, McCormack R, et al: Complications following management of displaced intra-articular calcaneal fractures: A prospective randomized trial comparing open reduction internal fixation with nonoperative management.  J Orthop Trauma 2003;17:241-249.

Buckley R, Tough S, McCormack R, et al: Operative compared with nonoperative treatment of displaced intra-articular calcaneal fractures: A prospective, randomized, controlled multicenter trial.  J Bone Joint Surg Am 2002;84:1733-1744.

 

26.    Figures 14a and 14b show the initial radiographs of an 18-year-old man who fell while snowboarding.  Figures 14c and 14d show the radiographs obtained following closed reduction.  Examination reveals that the elbow is stable with range of motion.  Management should now consist of

 

1-         immediate return to unrestricted activity.

2-         a posterior long arm splint for 7 to 10 days, followed by elbow range-of-motion exercises.

3-         a long arm cast for 4 weeks.

4-         immediate surgical repair of the collateral ligaments.

5-         immediate surgical repair of the collateral ligaments and placement of a hinged external fixator.

 

PREFERRED RESPONSE: 2

 

DISCUSSION: The initial radiographs reveal a simple elbow dislocation without associated fractures.  After successful closed reduction, the range of stability should be assessed.  If the elbow is stable, nonsurgical management should consist of a short period of immobilization followed by range-of-motion exercises.  Immobilization for more than 3 weeks results in significant elbow stiffness.  Surgical repair is indicated for dislocations that are irreducible, have associated fractures, or where stability cannot be maintained with closed treatment.

 

REFERENCES: Cohen MS, Hastings H II: Acute elbow dislocations: Evaluation and management.  J Am Acad Orthop Surg 1998;6:15-23.

O’Driscoll SW: Elbow dislocations, in Morrey BF (ed): The Elbow and Its Disorders, ed 3.  Philadelphia, PA, WB Saunders, 2000, pp 409-420. 

 

27.    A 12-year-old boy sustains open comminuted midshaft tibial and fibular fractures while playing indoor soccer.  The wound is grossly clean and measures 7 cm with some periosteal stripping.  Antibiotics and tetanus toxoid are administered immediately in the emergency department.  Following irrigation and debridement of the wound in the operating room, treatment should include

 

1-         a long leg cast.

2-         a reamed nail.

3-         an unreamed nail.

4-         an external fixator.

5-         plates and screws.

 

PREFERRED RESPONSE: 4

 

DISCUSSION: Open fractures in children have similar rates of short-term complications such as compartment syndrome, vascular injury, and nerve injury when compared to adult fractures.  Primary wound closure should be used for Gustillo and Anderson type 1 or uncomplicated type 2 fractures after surgical debridement.  Skeletal stabilization may consist of external fixation, flexible nails, or casting with or without supplementary pin fixation.  For an open comminuted midshaft fracture, external fixation is the treatment of choice.  Reamed intramedullary nailing is contraindicated in children with an open physis.  Plate fixation has a high complication rate in severe open fractures. 

 

REFERENCES: Jones BG, Duncan RD: Open tibial fractures in children under 13 years of age-10 years experience.  Injury 2003;34:776-780.

Bartlett CS III, Weiner LS, Yang EC: Treatment of type II and type III open tibia fractures in children.  J Orthop Trauma 1997;11:357-362.

Robertson P, Karol LA, Rab GT: Open fractures of the tibia and femur in children.  J Pediatr Orthop 1996;16:621-626.

Cullen MC, Roy DR, Crawford AH, et al: Open fracture of the tibia in children.  J Bone Joint Surg Am 1996;78:1039-1047.

 

28.    Which of the following is an advantage of unreamed nailing of the tibia compared to reamed nailing? 

 

1-         Less surgical time

2-         Lower risk of nonunion

3-         Lower rate of malunion

4-         Faster time to union

5-         Less secondary procedures to achieve union

 

PREFERRED RESPONSE: 1

 

DISCUSSION: The debate between reamed versus unreamed intramedullary nailing of the tibia continues.  Although unreamed nailing was proposed for open fractures to minimize infection, its simplicity made it appealing for closed fractures.  However, most studies to date show that the only advantage of unreamed nailing is less surgical time.  All studies show higher nonunion rates with increased hardware failure and increased time to union for unreamed nailing.  Even in open fractures graded up to Gustilo Grade IIIA, the reamed tibial nail performs better.

 

REFERENCES: Larsen LB, Madsen JE, Hoiness PR, et al: Should insertion of intramedullary nails for tibial fractures be with or without reaming?  A prospective, randomized study with 3.8 years’ follow-up.  J Orthop Trauma 2004;18:144-149.

Blachut PA, O’Brien PJ, Meek RN, et al: Interlocking intramedullary nailing with or without reaming for the treatment of closed fractures of the tibial shaft: A prospective randomized study.  J Bone Joint Surg Am 1997;79:640-646.

 

29.    A 12-year-old boy sustained a both bone forearm fracture 10 weeks ago and underwent closed reduction and casting.  Examination now reveals that the injury is healed, but he is unable to extend his little and ring fingers of the injured hand with his wrist extended.  Full extension is possible with the wrist flexed.  A radiograph and clinical photograph are shown in Figures 15a and 15b.  The remainder of his hand and wrist examination and neurologic evaluation in the hand are normal.  What is the most likely diagnosis?

 

1-         Unrecognized laceration of the extensor tendon to the ring and little fingers

2-         Unrecognized compartment syndrome

3-         Entrapment of the flexor digitorum profundus to the ring and little fingers

4-         Triggering at the A1 pulleys

5-         Ulnar nerve injury below the elbow

 

PREFERRED RESPONSE: 3

 

DISCUSSION: In this patient, examination reveals an inability to extend the fingers with the wrist extended, but full extension is possible with wrist flexion.  These findings demonstrate isolated tenodesis of the flexor digitorum to the ring and little fingers.  These findings are not consistent with compartment syndrome or nerve injury.  Scarring or entrapment of tendons in forearm fractures can occur.

 

REFERENCES: Watson PA, Blair W: Entrapment of the index flexor digitorum profundus tendon after fracture of both forearm bones in a child.  Iowa Orthop J 1999;19:127-128.

Shaw BA, Murphy KM: Flexor tendon entrapment in ulnar shaft fractures.  Clin Orthop 1996;330:181-184.

Kolkman KA, van Niekerk JL, Rieu PN, et al: A complicated forearm greenstick fracture: Case report.  J Trauma 1992;32:116-117.

Hendel D, Aner A: Entrapment of the flexor digitorum profundus of the ring finger at the site of an ulnar fracture: A case report.  Ital J Orthop Traumatol 1992;18:417-419.

 

30.    An otherwise healthy 35-year-old woman reports dorsal wrist pain and has trouble extending her thumb after sustaining a minimally displaced fracture of the distal radius 3 months ago.  What is the next most appropriate step in management?

 

1-         Neurophysiologic test to evaluate the posterior interosseous nerve

2-         Transfer of the extensor indicis proprius to the extensor pollicis longus tendon

3-         Interphalangeal joint arthrodesis of the thumb

4-         Extension splinting of the thumb

5-         Fine cut CT of the distal radius to evaluate Lister’s tubercle

 

PREFERRED RESPONSE: 2

 

DISCUSSION: Extensor pollicis longus tendon rupture can occur after a fracture of the distal radius, even a minimally displaced one.  Poor vascularity of the tendon within the third dorsal compartment is the suspected etiology, not the displaced fracture fragments.  Tendon transfer will suitably restore active extension of the thumb interphalangeal joint.

 

REFERENCES: Christophe K: Rupture of the extensor pollicis longus tendon following Colles fracture.  J Bone Joint Surg Am 1953;35:1003-1005.

Hove LM: Delayed rupture of the thumb extensor tendon: A 5-year study of 18 consecutive cases.  Acta Orthop Scand 1994;65:199-203.

 

31.     Figure 16a shows the radiograph of a 34-year-old woman who sustained a basicervical fracture of the femoral neck.  The fracture was treated with a compression screw and side plate.  Seven months postoperatively, she continues to have significant hip pain and cannot bear full weight on her hip.  A recent radiograph is shown in Figure 16b.  Management should now consist of

 

1-         continued non-weight-bearing and a bone stimulator.

2-         removal of the hardware, bone grafting of the femoral neck, and refixation.

3-         removal of the hardware and hemiarthroplasty.

4-         removal of the hardware and total hip arthroplasty.

5-         removal of the hardware and a valgus osteotomy.

 

PREFERRED RESPONSE: 5

 

DISCUSSION: The patient sustained a high-angle femoral neck fracture.  The follow-up clinical findings and radiograph show that she now has a nonunion with failed internal fixation.  The joint appears preserved.  In a healthy, young patient, arthroplasty of the femoral head, although possible, is not ideal.  Excellent healing and function can be obtained in 70% to 80% of patients with femoral neck nonunion with a valgus intertrochanteric osteotomy.

 

REFERENCES: Marti RK, Schuller HM, Raaymakers EL: Intertrochanteric osteotomy for non-union of the femoral neck.  J Bone Joint Surg Br 1989;71:782-787.

Ballmer FT, Ballmer PM, Baumgaertel F, et al: Pauwels osteotomy for nonunions of the femoral neck.  Orthop Clin North Am 1990;21:759-767.

 

32.     An 18-year-old man was in a motor vehicle accident and sustained a closed head injury, right displaced scapular body and glenoid fractures, a right proximal humeral fracture, fractures of ribs one through three, facial fractures, and bilateral pubic rami fractures with minimal displacement.  He has a systolic blood pressure of 80/40 mm Hg despite fluid resuscitation.  A radiograph is shown in Figure 17.  Spiral CT does not identify any thoracic or abdominal injuries.  What is the next most appropriate step in management?

 

1-         Pelvic angiography

2-         Intracranial pressure monitoring

3-         Pelvic external fixation

4-         Evaluation of peripheral pulses

5-         Urgent open stabilization of the clavicular and humeral fractures

 

PREFERRED RESPONSE: 4

 

DISCUSSION: The patient has sustained high-energy upper extremity and chest injuries.  He continues to remain hemodynamically unstable with no obvious thoracic or abdominal injury responsible for bleeding.  The pelvic fracture is unlikely to be causing significant bleeding.  A scapulothoracic dissociation and possible disruption of one of the great vessels of the upper extremity should be considered.  Evaluation of peripheral pulses or blood pressure indices bilaterally in the upper extremities is a simple way to evaluate the need for further work-up.  If there is any discrepancy or further concern, angiography of the involved extremity is necessary. 

 

REFERENCES: Althausen PL, Lee MA, Finkemeier CG: Scapulothoracic dissociation: Diagnosis and treatment.  Clin Orthop 2003;416:237-244.

Witz M, Korzets Z, Lehmann J: Traumatic scapulothoracic dissociation.  J Cardiovasc Surg 2000;41:927-929.

 

33.     What is the major difference in outcome following open reduction and internal fixation (ORIF) of the tibial plafond at 2 to 5 days versus 10 to 20 days?

 

1-         Improved ankle range of motion

2-         Increased risk of wound complications

3-         Decreased ankle pain

4-         Decreased risk of nerve injuries

5-         Decreased risk of development of traumatic arthritis

 

PREFERRED RESPONSE: 2

 

DISCUSSION: Long-term outcomes following tibial plafond fractures treated with ORIF are satisfactory in most patients despite a high incidence of posttraumatic osteoarthritis.  If ORIF is delayed until 10 to 20 days following injury, the major difference in outcomes is fewer complications associated with wound healing.  Ankle strength, pain, range of motion, and the development of arthritis are equal regardless of the time until fixation.

 

REFERENCES: Sirkin M, Sanders R, DePasquale T, et al: A staged protocol for soft tissue management in the treatment of complex pilon fractures.  J Orthop Trauma 1999;13:78-84.

Pollak AN, McCarthy ML, Bess RS, et al: Outcomes after treatment of high-energy tibial plafond fractures.  J Bone Joint Surg Am 2003;85:1893-1900.

 

34.     Figure 18a shows the initial lateral radiograph of a 6-year-old girl who sustained a fracture in a motor vehicle accident and was treated in a cast 1 year ago.  She now has the valgus deformity seen in Figure 18b.  Treatment should consist of

 

1-         observation.

2-         high tibial osteotomy.

3-         MRI and assessment for growth arrest and bar excision.

4-         stapling of the lateral tibial physis.

5-         external fixation and hemichondrodiastasis.

 

PREFERRED RESPONSE: 1

 

DISCUSSION: Proximal tibial metaphyseal fractures may result in late genu valgum as a result of asymmetric growth of the proximal tibia.  These patients are best treated with observation because the deformity is likely to remodel.  Osteotomy is not indicated and potentially will lead to recurrence.  Stapling of the medial tibial physis is appropriate in patients who have a severe and progressive deformity. 

 

REFERENCES: Cozen L: Knock-knee deformity in children: Congenital and acquired.  Clin Orthop 1990;258:191-203.

Jackson DW, Cozen L: Genu valgum as a complication of proximal tibial metaphyseal fractures in children.  J Bone Joint Surg Am 1971;53:1571-1578.

Brammar TJ, Rooker GD: Remodeling of valgus deformity secondary to proximal metaphyseal fracture of the tibia.  Injury 1998;29:558-560.

Ogden JA, Ogden DA, Pugh L, et al: Tibia valga after proximal metaphyseal fractures in childhood: A normal biologic response.  J Pediatr Orthop 1995;15:489-494.

Salter RB, Best TN: Pathogenesis of progressive valgus deformity following fractures of the proximal metaphyseal region of the tibia in young children.  Instr Course Lect 1992;41:409-411.

 

35.     Figure 19 shows the radiograph of a 45-year-old woman who has a painful nonunion.  Treatment should consist of

 

1-         revision internal fixation with a longer side plate and bone grafting.

2-         open reduction and internal fixation with a 95° fixed angle device and
bone grafting.

3-         hardware removal and retrograde intramedullary nailing.

4-         placement of an implantable bone stimulator.

5-         proximal femoral resection and total hip arthroplasty.

 

PREFERRED RESPONSE: 2

 

DISCUSSION: The radiograph reveals a reverse obliquely subtrochanteric/intertrochanteric fracture.  Open reduction and internal fixation should be accomplished with a 95° fixed angle device.  An intramedullary nail with screw fixation into the head is another possible technique.  Either method should correct the varus deformity.  Exchange of a high-angled screw and plate device to a longer side plate and bone grafting does not afford any improvement in the mechanical stability.  Hardware removal and retrograde intramedullary nailing is not indicated for this level of a proximal femoral injury.  Placement of an implantable bone stimulator may change local biologic factors but would not enhance mechanical stability.  The patient’s femoral head is intact without signs of collapse; therefore, hardware removal, proximal femoral resection, and total hip arthroplasty are not warranted.

 

REFERENCES: Haidukewych GJ, Israel TA, Berry DJ: Reverse obliquity fractures of the intertrochanteric region of the femur.  J Bone Joint Surg Am 2001;83:643-650.

Koval KJ, Zuckerman JD: Intertrochanteric fractures, in Rockwood & Green’s Fractures in Adults, ed 5.  Philadelphia, PA, Lippincott Williams and Wilkins, 2001, pp 1635-1681.

 

36.     A 7-year-old boy has a swollen and deformed right arm after falling off his bicycle.  Radiographs reveal a completely displaced posterolateral supracondylar humeral fracture.  Examination reveals a warm, pink hand and forearm but absent pulses.  What is the next most appropriate step in management?

 

1-         Angiography

2-         Immediate closed reduction and casting in extension

3-         Surgical exploration and repair of the artery, followed by skeletal  stabilization

4-         Closed reduction and pinning, followed by reassessment of the vascular status

5-         Magnetic resonance angiography (MRA)

 

PREFERRED RESPONSE: 4

 

DISCUSSION: The incidence of vascular injury in supracondylar humeral fractures is directly related to the degree and direction of displacement.  Significant posterior lateral displacement tends to result in brachial artery and median nerve injuries, and posterior medial displacement may lead to radial nerve injury.  The brachial artery is always injured at the level of the fracture; therefore, angiography or MRA will not assist in locating the injury.  The treatment of choice is surgical reduction and stabilization of the fracture, followed by reassessment of the vascular status.  If the hand is pink and warm or pulses can be detected with doppler, it is reasonable to follow the extremity closely after surgery.  If the arm becomes pulseless and white, immediate anterior exploration of the arm is indicated.  The artery is often entrapped in the fracture and once extricated, will provide adequate blood flow.  If the artery is injured, a primary repair or vein graft is needed.

 

REFERENCES: Shaw BA: The role of angiography in assessing vascular injuries associated with supracondylar humerus fractures remains controversial.  J Pediatr Orthop 1998;18:273. 

Sabharwal S, Tredwell SJ, Beauchamp RD, et al: Management of pulseless pink hand in pediatric supracondylar fractures of humerus.  J Pediatr Orthop 1997;17:303-310.

Schoenecker PL, Delgado E, Rotman M, et al: Pulseless arm in association with totally displaced supracondylar fracture.  J Orthop Trauma 1996;10:410-415.

 

37.    What is the treatment of choice for the injury shown in Figures 20a through 20c?

 

1-         Closed reduction and a short arm cast

2-         Splinting in a functional position and early motion

3-         Closed or open reduction and internal fixation with Kirschner wires

4-         Open reduction and internal fixation with mini-fragment screws

5-         Primary arthrodeses of the carpometacarpal joints

 

PREFERRED RESPONSE: 3

 

DISCUSSION: The radiographs show multiple carpometacarpal dislocations.  Reduction is often obtainable but difficult to maintain.  Internal fixation is required to maintain the reduction, preferably with Kirschner wires.  Closed reduction and percutaneous pinning is preferred by some surgeons.  Others recommend open reduction to remove irreconstructable osteochondral fragments from the individual joints and to ensure correct reduction of the carpometacarpal joints.  Kirschner wires are removed at 6 to 8 weeks.

 

REFERENCES: Prokuski LJ, Eglseder WA Jr: Concurrent dorsal dislocations and fracture-dislocations of the index, long, ring, and small (second to fifth) carpometacarpal joints.  J Orthop Trauma 2001;15:549-554.

Lawlis JF III, Gunther SF: Carpometacarpal dislocations: Long-term follow-up.  J Bone Joint Surg Am 1991;73:52-59.

 

38.    A 32-year-old man has intense right hand and wrist pain, a deformed wrist, and numbness in his fingers after falling off his motorcycle.  This is an isolated injury.  Examination reveals a swollen wrist, normal capillary refill to all fingers, and limited flexion of all fingers.  Radiographs are shown in Figures 21a and 21b.  Neurologic examination of the hand will most likely reveal

 

1-         lack of extension of the thumb.

2-         lack of abduction of the little finger.

3-         decreased sensation on the volar surface of the index finger.

4-         decreased sensation on the volar surface of the little finger.

5-         inability to extend the metacarpophalangeal joints of the fingers.

 

PREFERRED RESPONSE: 3

 

DISCUSSION: The patient has a perilunate dislocation.  A volar dislocation of the lunate is often associated with median nerve dysfunction.  This injury to the wrist is often overlooked because of its benign clinical appearance and the presence of other injuries, as it is caused by high-energy mechanisms.

 

REFERENCES: Ruby LK, Cassidy C: Fractures and dislocations of the carpus, in Browner BD (ed): Skeletal Trauma, ed 3.  Philadelphia, PA, WB Saunders, 2003, pp 1297-1300.

Habernek H, Weinstabl R, Kdolsky R, et al: Volar lunate fracture-dislocations of the wrist: Case report for two patients treated with external frame and internal open reduction.  J Trauma 1998;45:975-978.

 

39.    A 55-year-old woman fell and sustained an elbow dislocation with a coronoid fracture and a radial head fracture.  The elbow is reduced and splinted.  What is the most common early complication?

 

1-         Brachial artery intimal tear

2-         Recurrent dislocation

3-         Forearm compartment syndrome

4-         Posterior interosseous nerve injury

5-         Ulnar nerve palsy

 

PREFERRED RESPONSE: 2

 

DISCUSSION: The patient has a dislocation of the elbow with displaced coronoid process and radial head fractures.  The elbow is extremely unstable after this injury, and recurrent dislocation in a splint is the most common early complication.  Skeletal stabilization of the fractures is required to restore stability of the joint.  Characteristics of the fractures will determine the techniques required to restore stability.

 

REFERENCES: Ring D, Jupiter JB, Zilberfarb J: Posterior dislocation of the elbow with fractures of the radial head and coronoid.  J Bone Joint Surg Am 2002;84:547-551.

Ring D, Jupiter JB: Fracture-dislocation of the elbow.  J Bone Joint Surg Am 1998;80:566-580.

 

40.    A 25-year-old man sustained the closed injury shown in Figures 22a and 22b.  Examination reveals that this is an isolated injury, and he is hemodynamically stable.  Treatment should consist of

 

1-         multiple flexible intramedullary nails.

2-         unreamed intramedullary nailing with static interlocking.

3-         unreamed intramedullary nailing with dynamic interlocking.

4-         reamed intramedullary nailing with static interlocking.

5-         reamed intramedullary nailing with dynamic interlocking.

 

PREFERRED RESPONSE: 4

 

DISCUSSION: The treatment of choice for closed diaphyseal femoral fractures in adults is reamed intramedullary nailing with static interlocking.  Reaming allows placement of a larger, stronger implant and offers better healing rates than unreamed nailing.  Static interlocking ensures that there is no loss of reduction because of underappreciated fracture lines or comminution.

 

REFERENCES: Brumback RJ, Virkus WW: Intramedullary nailing of the femur: Reamed versus nonreamed.  J Am Acad Orthop Surg 2000;8:83-90.

Brumback RJ, Ellison TS, Poka A, et al: Intramedullary nailing of femoral shaft fractures: Part III. Long-term effects of static interlocking fixation.  J Bone Joint Surg Am 1992;74:106-112.

 

41.     Figure 23 shows the radiograph of an elderly man who fell on his right arm.  What is the most important determinate of a good outcome following this injury?

 

1-         Early open reduction and internal fixation

2-         Initiation of physical therapy and passive motion within 2 weeks of the injury

3-         Fracture involvement of the greater tuberosity

4-         Immobilization with a sling and swathe for 4 weeks

5-         Age of younger than 70 years

 

PREFERRED RESPONSE: 2

 

DISCUSSION: Minimally displaced fractures of the proximal humerus have a good outcome if physical therapy is initiated within 2 weeks of the injury.  Results are not affected by age, open reduction and internal fixation, or involvement of the greater tuberosity.  Immobilization for longer than 3 weeks will often result in stiffness.

 

REFERENCES: Koval KJ, Gallagher MA, Marsicano JG, et al: Functional outcome after minimally displaced fractures of the proximal part of the humerus.  J Bone Joint Surg Am 1997;79:203-207.

Hodgson SA, Mawson SJ, Stanley D: Rehabilitation after two-part fractures of the neck of the humerus.  J Bone Joint Surg Br 2003;85:419-422.

 

42.    A 40 year-old-man was involved in a motor vehicle accident and sustained the pelvic injury seen in Figures 24a and 24b.  Definitive management of the injury should consist of reduction by

 

1-         skeletal traction and bed rest.

2-         anterior external fixation.

3-         internal fixation of the symphysis pubis.

4-         internal fixation of the symphysis pubis with supplemental external fixation.

5-         internal fixation of the symphysis pubis and sacral fracture.

 

PREFERRED RESPONSE: 5

 

DISCUSSION: The radiograph reveals disruption of the symphysis pubis and a displaced left sacral fracture.  A posterior injury with displacement of greater than 1 cm is unstable, and a sacral fracture is particularly unstable.  Surgical stabilization is required for these unstable anterior and posterior injuries.  External fixation provides little stability to an unstable posterior pelvic injury. Reduction and internal fixation of the symphysis pubis and sacral fracture will provide the most stable pelvis with the least resultant deformity and allow patient mobilization.

 

REFERENCES: Tile M: Management of pelvic ring injuries, in Tile M, Helfet DL, Kellam JF (eds): Fractures of the Pelvis and Acetabulum, ed 3.  Philadelphia, PA, Lippincott Williams & Wilkins, 2003, pp 168-202.

Kabak S, Halici M, Tuncel M, et al: Functional outcome of open reduction and internal fixation for completely unstable pelvic ring fractures (type C): A report of 40 cases.  J Orthop Trauma 2003;17:555-562.

 

43.     A 35-year-old patient sustained a bimalleolar ankle fracture.  What is the most reliable method of predicting a tear of the interosseous membrane?

 

1-         Level of the fibular fracture

2-         Lauge-Hansen fracture class

3-         Intraoperative stress testing

4-         Widening of the medial clear space

5-         Talar dislocation

 

PREFERRED RESPONSE: 3

 

DISCUSSION: The Weber and Lauge-Hansen fracture classifications suggest that the interosseous membrane (IOM) is torn with certain fracture patterns.  In a recent study that evaluated ankle fractures with MRI, Nielson and associates identified 30 patients with IOM tears.  Ten of the tears did not correspond with the level of the fibular fracture.  The authors concluded that stability of the syndesmosis should not be based on the level of the fibular fracture alone but should also include an intraoperative stress test.  Transsyndesmotic fixation should be considered for those fractures where the intraoperative stress test demonstrates instability.  A widened medial clear space may occur with a deltoid injury and distal fibular fracture in the absence of a significant tear of the interosseous membrane.

 

REFERENCE: Nielson JH, Sallis JG, Potter HG, et al: Correlation of interosseous membrane tears to the level of the fibular fracture.  J Orthop Trauma 2004;18:68-74.

 

44.    When performing a flexor tendon repair of a digit other than the thumb, what structures of the flexor tendon sheath should be preserved?

 

1-         A1 and A2 pulleys

2-         A1 and A3 pulleys

3-         A2 and A3 pulleys

4-         A2 and A4 pulleys

5-         C1 and C2 pulleys

 

PREFERRED RESPONSE: 4

 

DISCUSSION: The A2 and A4 pulleys are considered the most important parts of the pulley system.  If these two structures are preserved, 80% of finger flexion can be maintained.  If the pulley system is not left intact or is not reconstructed, “bow-stringing” of the flexor tendons occurs with loss of full flexion.  The A2 pulley is over the proximal phalanx and the A4 pulley is over the middle phalanx.

 

REFERENCES: Doyle JR: Anatomy of the finger flexor tendon sheath and pulley system. 
J Hand Surg Am 1988;13:473-484.

Strickland JW: Flexor tendon injuries: I. Foundations of treatment.  J Am Acad Orthop Surg 1995;3:44-54.

 

45.    A distal radius fracture in an elderly man is strongly predictive for what
subsequent injury?

 

1-         Another distal radius fracture

2-         Insufficiency fracture of the spine

3-         Insufficiency fracture of the pelvis

4-         Hip fracture

5-         Proximal humerus fracture

 

PREFERRED RESPONSE: 4

 

DISCUSSION: Fractures of the distal radius increase the relative risk of a subsequent hip fracture significantly more in men than in women.  A previous spinal fracture has an equally important impact on the risk of a subsequent hip fracture in both genders. 

 

REFERENCE: Haentjens P, Autier P, Collins J, et al: Colles fracture, spine fracture, and subsequent risk of hip fracture in men and women: A meta-analysis.  J Bone Joint Surg Am 2003;85:1936-1943.

 

46.    A 13-year-old girl injures her ankle playing soccer.  Radiographs reveal a displaced Tillaux fracture.  CT scans are shown in Figure 25.  What is the most important consideration for appropriate management?

 

1-         Joint congruity

2-         Torn anterior tibiofibular ligament

3-         Growth arrest leading to angular deformity

4-         Growth arrest leading to limb-length discrepancy

5-         Osteonecrosis of the talus

 

PREFERRED RESPONSE: 1

 

DISCUSSION: Tillaux and triplane fractures occur in adolescents as the result of an external rotation injury of the ankle.  As seen on the CT scan, the growth plate starts to close during adolescence; therefore, growth arrest resulting in limb-length discrepancy or angulation is less of a concern in this age group than achieving joint congruity.  The joint should be surgically reduced if displacement is greater than 2 mm to minimize the chances of late arthrosis.

 

REFERENCES: Kay RM, Matthys GA: Pediatric ankle fractures: Evaluation and treatment. 
J Am Acad Orthop Surg 2001;9:268-278.

Kling TF Jr: Operative treatment of ankle fractures in children.  Orthop Clin North Am 1990;21:381-392.

Duchesneau S, Fallat LM: The Tillaux fracture.  J Foot Ankle Surg 1996;35:127-133.

 

47.    What measure of physiologic status best evaluates whether an injured patient is fully resuscitated and best predicts that perioperative complications will be minimized following definitive stabilization of long bone fractures?

 

1-         Urine output of greater than 100 mL/h

2-         Cardiac output of greater than 2

3-         Serum lactate level of less than 2.5 mmol/L

4-         Systolic blood pressure of greater than 100 mm Hg

5-         Hemoglobin level of greater than 10 g/dL 

 

PREFERRED RESPONSE: 3

 

DISCUSSION: Serum lactate levels can be used to evaluate the effectiveness of the resuscitation of patients who have multiple injuries.  Even after resuscitation, patients may have occult hypoperfusion as defined by a serum lactate level of greater than 2.5 mmol/L.  The studies referenced indicate that these patients are at increased risk of perioperative complications such as organ failure or adult respiratory distress syndrome if definitive surgical fixation of the orthopaedic injuries is pursued prior to correction of the occult hypoperfusion.  The other markers may be an indication of current physiology but have not been correlated with perioperative risks. 

 

REFERENCES: Blow O, Magliore L, Claridge JA, et al: The golden hour and silver day: Detection and correction of occult hypoperfusion within 24 hours improves outcomes from major trauma.  J Trauma 1999;47:964-977.

Crowl A, Young JS, Kahler DM, et al: Occult hypoperfusion is associated with increased morbidity in patients undergoing early femur fracture fixation.  J Trauma 2000;48:260-267.

Shulman AM: Prediction of patients who will develop prolonged occult hypoperfusion following blunt trauma.   J Trauma 2004;57:725-800.

 

48.    Based on the findings seen in the radiograph in Figure 26, emergent management should consist of

 

1-         volar open reduction and pinning.

2-         open reduction, ligament repair, and pinning.

3-         closed reduction and splinting.

4-         closed reduction and pinning.

5-         dorsal open reduction and pinning.

 

PREFERRED RESPONSE: 3

 

DISCUSSION: The radiograph shows a volarly dislocated lunate.  Initial emergent treatment of perilunate dislocations should consist of closed reduction and splinting, especially if the patient exhibits median nerve compression.  Open reduction and pinning or ligament repair are necessary but are not emergent.  A dorsal approach is sometimes required for ligament repair or bony visualization; however, this can be done in a more semi-elective manner. 

 

REFERENCES: Isenberg J, Prokop A, Schellhammer F, et al: Palmar lunate dislocation.  Unfallchirurg 2002;105:1133-1138.

Ruby LK: Fractures and dislocations of the carpus, in Browner BD, Jupiter JB (eds): Skeletal Trauma, ed 2.  Philadelphia, PA, WB Saunders, 1998, pp 1367-1372.

 

49.    A 10-year-old girl has a midshaft both bone forearm fracture.  After attempted closed reduction, alignment consists of bayonet apposition, 10° of malrotation, and 8° of volar angulation.  Management should now consist of

 

1-         open reduction and plating of the radius to restore the radial bow.

2-         open reduction of the ulna and plating.

3-         closed reduction and nailing of the bones with flexible nails.

4-         a long arm cast and follow-up of alignment in 5 days.

5-         a short arm cast for 6 weeks.

 

PREFERRED RESPONSE: 4

 

DISCUSSION: Acceptable alignment in both bone forearm fractures is related to age and location.  In children younger than age 9 years, angulations of 15° and malrotation of 45° are acceptable.  In children older than age 9 years, acceptable alignment is 10° of angulation and 30° of malrotation.  Bayonet apposition is acceptable provided that the angular and rotational reductions are held within these guidelines.  A long arm cast provides better control of deforming forces than a short arm cast.

 

REFERENCES: Do TT, Strub WM, Foad SL, et al: Reduction versus remodeling in pediatric distal forearm fractures: A preliminary cost analysis.  J Pediatr Orthop B 2003;12:109-115.

Flynn JM: Pediatric forearm fractures: Decision making, surgical techniques, and complications.  Instr Course Lect 2002;51:355-360.

Ring D, Waters PM, Hotchkiss RN, et al: Pediatric floating elbow.  J Pediatr Orthop 2001;21:456-459.

Noonan KJ, Price CT: Forearm and distal radius fractures in children.  J Am Acad Orthop Surg 1998;6:146-156.

 

50.    In the treatment of ankle fractures, the superficial peroneal nerve is most commonly injured by

 

1-         a posterior-lateral approach.

2-         a lateral approach.

3-         a medial approach.

4-         an anterior-medial approach.

5-         rigid cast immobilization.

 

PREFERRED RESPONSE: 2

 

DISCUSSION: In the treatment of ankle fractures, the superficial peroneal nerve is most commonly injured by the use of a direct lateral approach to the ankle.  The superficial peroneal nerve and its branches exit the fascial hiatus approximately 9 cm to 10 cm proximal to the tip of the distal fibula with a range of 4 cm to 13 cm, and their course is typically anterior to the midlateral plane of the fibula.  However, small branches may course across the surgical plane directly laterally.  A posterior-lateral approach diminishes the risk of injury to the superficial peroneal nerve and its branches; however, by moving farther posterior, the sural nerve and its branches may be at increased risk.  Cast immobilization may injure the cutaneous nerves about the ankle; however, the risks are greater with surgical intervention.  A medial or anterior-medial approach to the ankle will not injure the superficial peroneal nerve at the ankle level.

 

REFERENCES: Redfern DJ, Sauve PS, Sakellariou A: Investigation of incidence of superficial peroneal nerve injury following ankle fracture.  Foot Ankle Int 2003;24:771-774.

Miller SD: Ankle fractures, in Myerson MS (ed): Foot and Ankle Disorders.  Philadelphia, PA, WB Saunders, 2000, pp 1341-1366.

 

51.     A 54-year-old man sustained a small superficial abrasion over the left acromioclavicular joint after falling from his bicycle.  Examination reveals no other physical findings.  Radiographs show a displaced fracture of the lateral end of the clavicle distal to a line drawn vertically to the coracoid process.  Management should consist of

 

1-         open reduction and plate fixation.

2-         a figure-of-8 bandage for 4 to 6 weeks.

3-         a sling for comfort, followed by physical therapy when pain-free.

4-         excision of the outer end of the clavicle.

5-         a tension band and Kirschner wires.

 

PREFERRED RESPONSE: 3

 

DISCUSSION: Displaced clavicular fractures lateral to the coracoid process (Neer type II and III) are best managed nonsurgically with sling immobilization and physical therapy, starting with pendulum exercises and progressing to active-assisted exercises when comfortable.  Supervised therapy should be performed for 3 months or until full painless motion is achieved.  In one study by Robinson and Cairns, this form of treatment provided patients with a 86% chance of avoiding a secondary reconstructive procedure. 

 

REFERENCES: Robinson CM, Cairns DA: Primary nonoperative treatment of displaced lateral fractures of the clavicle.  J Bone Joint Surg Am 2004;86:778-782.

Deafenbaugh MK, Dugdale TW, Staeheli JW, et al: Nonoperative treatment of Neer type II distal clavicle fractures: A prospective study.  Contemp Orthop 1990;20:405-413.

 

52.    A 47-year-old man sustained a degloving injury over the pretibial surface and anterior ankle region in a motor vehicle accident.  After debridement and irrigation, there is inadequate tissue for closure of the exposed anterior tibial tendon and tibia.  Prior to definitive soft-tissue coverage, management should consist of

 

1-         immediate split-thickness skin grafting.

2-         immediate Xenograft application.

3-         a vacuum-assisted closure device.

4-         dressing changes with sulfasalazine cream.

5-         a cross-leg flap.

 

PREFERRED RESPONSE: 3

 

DISCUSSION: With soft-tissue loss, local or free flap coverage may be necessary to treat exposed tendon and bone.  However, a vacuum-assisted closure device is a good temporizing dressing.  It prevents external contamination, reduces edema around the wound, increases oxygen tension in the wound, and promotes the formation of granulation tissue.  The use of this negative pressure device has been described in both acute traumatic and in chronic wound scenarios.  If sufficient granulation tissue forms, closure may be by split graft, avoiding a more complex coverage procedure.  Immediate skin grafting over the exposed anterior tibial tendon and tibia would have a low likelihood of success.  Dressing changes with sulfasalazine may be beneficial in a burn wound to assist with removal of skin slough; however, in a granulating wound, the material may be toxic to early epithelialization.  Xenograft is a foreign body and should not be applied to an acute contaminated open wound.  Historically, a cross-leg flap was a treatment alternative for lower extremity soft-tissue loss; however, its current applications are extremely limited.

 

REFERENCES: Webb LX: New techniques in wound management: Vacuum assisted wound closure.  J Am Acad Orthop Surg 2002;10:303-311.

Clare MP, Fitzgibbons TC, McMullen ST, et al: Experience with the vacuum assisted closure negative pressure technique in the treatment of non-healing diabetic and dysvascular wounds.  Foot Ankle Int 2002;23:896-901.

 

53.     The humeral nonunion shown in Figure 27 is most likely to unite when using what method of treatment?

 

1-         Intramedullary nail

2-         Pulsed electromagnetic fields

3-         Compression plate

4-         Intramedullary nail and bone graft

5-         Compression plate and bone graft

 

PREFERRED RESPONSE: 5

 

DISCUSSION: The radiograph shows an atrophic nonunion of the humeral shaft.  The management of humeral nonunions has been studied with compression plates and bone graft, as well as intramedullary nailing and bone graft.  Compression plating with bone graft results in the highest rate of union.  Compression plating by itself is not adequate, given the bone loss and lack of callous in this nonunion.  Pulsed electromagnetic fields is a viable option for hypertrophic nonunions where there is inherent stability.  Intramedullary nailing does not provide as much compression and stability as that achieved with compression plating.

 

REFERENCES: Pugh DM, McKee MD: Advances in the management of humeral nonunion. 
J Am Acad Orthop Surg 2003;11:48-59.

McKee MD, Miranda MA, Riemer BL, et al: Management of humeral nonunion after the failure of locking intramedullary nails.  J Orthop Trauma 1996;10:492-499.

 

54.    An adult with a distal humeral fracture underwent open reduction and internal fixation.  What is the most common postoperative complication?

 

1-         Loss of elbow range of motion

2-         Nonunion

3-         Malunion

4-         Infection

5-         Ulnar nerve dysfunction

 

PREFERRED RESPONSE: 1

 

DISCUSSION: Most patients lose elbow range of motion after open reduction and internal fixation of a distal humeral fracture. Ulnar nerve dysfunction, nonunion, and infection all occur less commonly.

 

REFERENCES: Webb LX: Distal humerus fractures in adults.  J Am Acad Orthop Surg 1996;4:336-344.

McKee MD, Wilson TL, Winston L, et al: Functional outcome following surgical treatment of intra-articular distal humeral fractures through a posterior approach.  J Bone Joint Surg Am 2000;82:1701-1707.

 

55.    The radiographs and CT scan seen in Figures 28a through 28d reveal what type of acetabular fracture pattern?

 

1-         Transverse

2-         Transverse with posterior wall

3-         Both column

4-         Posterior wall anterior hemitransverse

5-         T-type

 

PREFERRED RESPONSE: 2

 

DISCUSSION: The AP, obturator oblique, and iliac oblique views of the pelvis reveal a fracture that disrupts the iliopectineal and ilioischial lines, indicating a fracture that involves both anterior and posterior columns.  However, it does not have the other features of anterior or posterior column fracture patterns.  A displaced posterior wall fracture is also present, best seen on the obturator oblique view.  The anterior to posterior directed fracture line on the CT scan indicates a transverse fracture; therefore, the patient has a transverse with posterior wall fracture pattern.  A T-type fracture would be similar but would have a break into the obturator ring.

 

REFERENCES: Tile M: Describing the injury: Classification of acetabular fractures, in Tile M, Helfet DL, Kellam JF (eds): Fractures of the Pelvis and Acetabulum, ed 3.  Philadelphia, PA, Lippincott Williams & Wilkins, 2003, pp 427-475.

Brandser E, Marsh JL: Acetabular fractures: Easier classification with a systematic approach.  Am J Roentgenol 1998;171:1217-1228.

 

56.    A 26-year-old man sustained an isolated injury to his left hip joint in a motor vehicle accident.  Closed reduction was performed, and the postreduction radiograph is shown in Figure 29.  Management should now consist of

 

1-         emergent open reduction and fixation of the fracture.

2-         skeletal traction and expedient open reduction and fixation of the fracture.

3-         skeletal traction for 6 weeks, followed by physical therapy.

4-         crutches and no weight bearing for 6 weeks.

5-         bed rest for 1 week and follow-up radiographs to see if the fragment has moved.

 

PREFERRED RESPONSE: 2

 

DISCUSSION: The patient has a posterior fracture-dislocation of the hip and following reduction, an incarcerated fragment of bone resulted in an incongruent reduction.  Whereas expedient removal of the fragment is required to limit articular cartilage damage, this situation is not an emergency and the procedure may be performed when the appropriate surgical team is available and the patient is stabilized.  Skeletal traction through either the femur or tibia may relieve some pressure on the joint and prevent articular damage.  Nonsurgical care for incarcerated fragments is contraindicated.

 

REFERENCES: Tile M, Olson SA: Decision making: Non operative and operative indications for acetabular fractures, in Tile M, Helfet DL, Kellam JF (eds): Fractures of the Pelvis and Acetabulum.  Philadelphia, PA, Lippincott Williams and Wilkins, 2003, pp 496-532.

Letournel E, Judet R: Fractures of the Acetabulum, ed 2.  Berlin, Germany, Springer Verlag, 1993, pp 337-339, p 507.

 

57.    A 35-year-old man is brought to the emergency department following a motorcycle accident.  He is breathing spontaneously and has a systolic blood pressure of 80 mm Hg, a pulse rate of 120/min, and a temperature of 98.6° F (37° C).  Examination suggests an unstable pelvic fracture; AP radiographs confirm an open book injury with vertical displacement on the left side.  Ultrasound evaluation of the abdomen is negative.  Despite administration of 4 L of normal saline solution, he still has a systolic pressure of 90 mm Hg and a pulse rate of 110.  Urine output has been about 20 mL since arrival 35 minutes ago.  What is the next best course of action?

 

1-         Continued resuscitation with fluids and blood

2-         Ongoing resuscitation and pelvic angiography

3-         Application of an external fixator in the emergency department

4-         A pelvic binder and continued resuscitation

5-         A pelvic binder, skeletal traction, and continued resuscitation

 

PREFERRED RESPONSE: 5

 

DISCUSSION: The patient is at risk for a pelvic vascular injury and major hemorrhage.  This type of complication of pelvic trauma is highest in motorcyclists.  Once it is recognized that the pelvic ring has opened, it is important to close that ring to tamponade any venous bleeding with a pelvic binder and to add a skeletal traction pin to the limb on the involved side.  This will correct any translational displacement.  The noninvasive pelvic binders or sheets are easy to apply and are very effective.  They do not compromise future care and allow the surgeons access to the abdomen.  External fixation or pelvic resuscitation clamps require a certain amount of skill to apply and are not always available.  If the pelvic stabilization does not improve the hemodynamic parameters in 10 to 15 minutes, angiography is necessary.

 

REFERENCE: Mayo K, Kellam JK: Pelvic ring disruptions, in Browner BD (ed): Skeletal Trauma, ed 3.  Philadelphia, PA, WB Saunders, 2003, pp 1052-1108.

 

58.    A healthy 25-year-old man sustains a grade IIIB open tibial fracture.  Following appropriate debridement, irrigation, and stabilization with an external fixator, the soft-tissue injury is shown in Figure 30.  What is the most appropriate definitive soft-tissue coverage procedure?

 

1-         Split-thickness skin graft

2-         Full-thickness skin graft

3-         Soleus rotation flap

4-         Medial gastrocnemius rotation flap

5-         Free latissimus dorsi flap with microvascular anastomosis

 

PREFERRED RESPONSE: 5

 

DISCUSSION: This is a very large near circumferential defect with posterior as well as anterior skin and muscle injury.  Bone is exposed.  The posterior muscles cannot be rotated since they are part of the zone of injury.  The bone and other poorly vascularized areas of this wound would not accept a skin graft.  The best chance for limb salvage will be to obtain soft-tissue coverage with a free tissue transfer using the latissimus dorsi.

 

REFERENCES: Mathes SJ, Nahai F: Vascular anatomy of muscle: Classification and applications, in Mathes SJ, Nahai F (eds): Clinical Application for Muscle and Musculocutaneous Flaps.  St Louis, MO, CV Mosby, 1982, p 20.

Bos GD, Buehler MJ: Lower-extremity local flaps.  J Am Acad Orthop Surg 1994;2:342-351.

 

59.    A 25-year-old woman undergoes surgical treatment of a displaced proximal humeral fracture via a deltopectoral approach.  At the first postoperative visit, she reports a tingling numbness along the anterolateral aspect of the forearm.  What structure is most likely injured?

 

1-         Medial cord of the brachial plexus

2-         Radial nerve

3-         Median nerve

4-         Axillary nerve

5-         Musculocutaneous nerve

 

PREFERRED RESPONSE: 5

 

DISCUSSION: Sensation along the anterolateral aspect of the forearm is supplied by the lateral antebrachial cutaneous nerve, the terminal branch of the musculocutaneous nerve.  The musculocutaneous nerve can be injured by proximal humeral fractures or dislocations, and is also at risk during surgical exposure if excessive retraction is placed on the conjoint tendon.  The musculocutaneous nerve enters the conjoint tendon 1 cm to 5 cm distal to the coracoid process.

 

REFERENCES: McIlveen SJ, Duralde XA, D’Alessandro DF, et al: Isolated nerve injuries about the shoulder.  Clin Orthop 1994;306:54-63.

Warner JP: Frozen shoulder: Diagnosis and management.  J Am Acad Orthop Surg
1997;5:130-140.

 

60.    A 7-year-old girl has pain and swelling of the right elbow after falling off her bicycle.  Radiographs are shown in Figure 31.  What is the most appropriate initial step in management?

 

1-         Cast immobilization in the current position for 6 weeks

2-         Closed reduction and cast immobilization for 6 weeks

3-         Reduction and internal fixation with Kirschner wires

4-         Arthrography to assess articular surface congruity

5-         MRI to assess articular congruity

 

PREFERRED RESPONSE: 3

 

DISCUSSION: Lateral condylar fractures are challenging to treat because of late displacement and development of a nonunion that may lead to valgus instability, pain, or tardy ulnar nerve palsy.  Fractures such as this one with more than 2 mm of displacement on any radiographic view are prone to nonunion and should be stabilized.  Fractures with less than 2 mm of displacement usually are stable and may be treated nonsurgically.  In these patients, careful follow-up is recommended within several days of casting to check for fracture displacement.  Arthrography or MRI may be helpful in these minimally displaced fractures.  Fractures with an intact articular cartilage surface, such as noted on these studies, are unlikely to displace further.

 

REFERENCES: Finnbogason T, Karlsson G, Lindberg L, et al: Nondisplaced and minimally displaced fractures of the lateral humeral condyle in children: A prospective radiographic investigation of fracture stability.  J Pediatr Orthop 1995;15:422-425.

Attarian DE: Lateral condyle fractures: Missed diagnoses in pediatric elbow injuries.  Mil Med 1990;155:433-434.

Flynn JC: Nonunion of slightly displaced fractures of the lateral humeral condyle in children: An update.  J Pediatr Orthop 1989;9:691-696.

Badelon O, Bensahel H, Mazda K, et al: Lateral humeral condylar fractures in children: A report of 47 cases. J Pediatr Orthop 1988;8:31-34.

 

61.     A 32-year-old man sustained a fracture of his upper arm in a motor vehicle accident.  Radiographs are shown in Figure 32.  Because of other associated injuries, surgical stabilization is chosen.  What technique will result in the least complications and the
best outcome?

 

1-         Retrograde locked intramedullary nail

2-         Antegrade reamed locked intramedullary nail

3-         Flexible nails

4-         Open reduction and plate fixation

5-         External fixation

 

PREFERRED RESPONSE: 4

 

DISCUSSION: Most humeral fractures will heal with nonsurgical functional brace management.  When the initial pain has subsided in a coaptation splint, the patient is converted to a functional brace and allowed to use the arm for activities.  The fracture should heal within 6 weeks to 12 weeks with acceptable results.  Surgery is indicated if there is vascular injury, open injury, floating elbow, chest injury, bilateral humeral fractures, or if a reduction cannot be obtained or maintained.  The surgical treatment of choice is either antegrade reamed locked intramedullary nailing or plate osteosynthesis.  Plate osteosynthesis appears to offer better results with respect to union, function, and risk of complications.

 

REFERENCES: Schemitsch EH, Bhandari M: Fractures of the humeral shaft, in Browner BD: Skeletal Trauma, ed 3.  Philadelphia, PA, WB Saunders, 2003, pp 1481-1511.

Chapman JR, Henley MB, Agel J: Randomized prospective study of humeral shaft fracture fixation: Intramedullary nails versus plates.  J Orthop Trauma  2000;14:162-166.

 

62.    A 56-year-old man sustained a nondisplaced extra-articular fracture of the proximal aspect of the third metatarsal after dropping a heavy object on his left foot.  Management should consist of

 

1-         open reduction and internal fixation.

2-         external bone stimulation.

3-         percutaneous pin fixation.

4-         weight bearing in a walking boot or walking cast.

5-         open reduction and internal fixation and primary tarsometatarsal joint fusion.

 

PREFERRED RESPONSE: 4

 

DISCUSSION: This injury pattern is one of a direct trauma to the mid aspect of the foot.  Without additional forces involved, capsular ligamentous injury is not anticipated; therefore, the injury should be a stable pattern.  Treatment should consist of protected weight bearing as tolerated in a walking boot or walking cast.  Surgical intervention with open reduction and internal fixation, percutaneous pinning, or open reduction and internal fixation with primary tarsometatarsal joint fusion is not indicated with this pattern of injury.  The use of external bone stimulation in this acute fracture setting is not indicated.  With injuries to the midfoot area where the exact mechanism of injury is uncertain, there should be a high index of suspicion for an associated injury to the tarsometatarsal joint, and standing radiographs or stress radiographs should be obtained. 

 

REFERENCES: Myerson MS: Foot and Ankle Disorders.  Philadelphia, PA, WB Saunders, 2000, pp 1265-1296.

Early JS: Fractures and dislocations of the midfoot and forefoot, in Rockwood and Green’s Fractures in Adults, ed 5.  Philadelphia, PA, Lippincott Williams and Wilkins, 2001,
pp 2181-2245.

 

63.     During a posterior approach to the glenoid with retraction as shown in Figure 33,
care should be taken during superior retraction to avoid injury to which of the
following structures?

 

1-         Axillary artery

2-         Axillary nerve

3-         Branch of the circumflex scapular artery

4-         Profunda brachii artery

5-         Suprascapular nerve and artery

 

PREFERRED RESPONSE: 5

 

DISCUSSION: During a posterior approach to the shoulder for either a scapular fracture,
glenoid fracture, or posterior shoulder pathology, the interval between the teres minor and infraspinatus is split.  Excessive superior retraction on the infraspinatus, or excessive dissection superomedially under the infraspinatus muscle and tendon can cause injury to the suprascapular nerve and/or artery.  During dissection in this interval, the axillary artery and axillary nerve are well protected.  A branch of the circumflex scapular artery ascends between the teres minor
and infraspinatus muscle, but it is at risk during dissection on the scapula in the mid portion of the interval and not during superior retraction.  The profunda brachii artery is not present in
this interval. 

 

REFERENCES: Jerosch JJ, Greig M, Peuker ET, et al: The posterior subdeltoid approach: A modified access to the posterior glenohumeral joint.  J Shoulder Elbow Surg 2001;10:265-268.

Judet R: Surgical treatment of scapular fractures.  Acta Orthop Belg 1964;30:673-678.

Kavanagh BF, Bradway JK, Cofield RH: Open reduction and internal fixation of displaced intra-articular fractures of the glenoid fossa.  J Bone Joint Surg Am 1993;75:479-484.

 

64.    A 42-year-old woman sustained a closed, displaced talar neck fracture in a motor vehicle accident.  Which of the following is an avoidable complication of surgical treatment?

 

1-         Posttraumatic arthritis of the subtalar joint

2-         Posttraumatic arthritis of the ankle joint

3-         Malunion of the talus

4-         Osteonecrosis of the talus

5-         Complex regional pain syndrome

 

PREFERRED RESPONSE: 3

 

DISCUSSION: Malunion of the talus is a devastating complication that leads to malpositioning of the foot and subsequent arthrosis of the subtalar joint complex.  This is considered an avoidable complication in that accurate surgical reduction will minimize its development.  Posttraumatic arthritis of the subtalar joint, osteonecrosis of the talus, posttraumatic arthritis of the ankle joint, and complex regional pain syndrome all may develop as a result of the initial traumatic event and may not be avoidable despite anatomic reduction.

 

REFERENCES: Rockwood and Green’s Fractures in Adults, ed 5.  Philadelphia, PA, Lippincott, Williams and Wilkins, 2001, pp 2091-2132.

Daniels TR, Smith JW, Ross TI: Varus malalignment of the talar neck: Its affects on the position of the foot and on subtalar motion.  J Bone Joint Surg Am 1996;78:1559-1567.

 

65.    Figures 34a through 34c show the radiographs of a 51-year-old woman who injured her elbow in a fall from standing height.  Examination reveals that elbow range of motion is limited by pain only.  Management should consist of

 

1-         open reduction and internal fixation.

2-         excision of the radial head.

3-         excision of the radial head and prosthetic arthroplasty.

4-         a long arm cast.

5-         a sling and early range-of-motion exercises.

 

PREFERRED RESPONSE: 5

 

DISCUSSION: The radiographs show a small minimally displaced radial head fracture that is amenable to nonsurgical management.  Early range-of-motion exercises will best restore
function and minimize stiffness.  A long arm cast for any length of time will result in severe elbow stiffness.

 

REFERENCES: Morrey BF: Radial head fracture, in Morrey BF (ed):  The Elbow and Its Disorders, ed 3.  Philadelphia, PA, WB Saunders, 2000, pp 341-364. 

Hotchkiss RN: Displaced fractures of the radial head: Internal fixation or excision?  J Am Acad Orthop Surg 1997;5:1-10.

 

66.    Figure 35 shows the radiograph of a 12-year-old boy who fell off a snowmobile and landed on his left shoulder.  He has a closed injury.  Management should consist of

 

1-         a shoulder spica cast.

2-         closed reduction and percutaneous pinning.

3-         open reduction to remove the interposed soft tissue.

4-         a shoulder sling, followed by repeat radiographs to document fracture position.

5-         open reduction and internal fixation with a 90° blade plate.

 

PREFERRED RESPONSE: 4

 

DISCUSSION: Proximal humeral fractures in children are classified as metaphyseal or Salter-Harris type I or II fractures, and most of these fractures are treated with closed methods.  Eighty percent of the growth of the humerus comes from the proximal physis; therefore, tremendous remodeling potential is present.  Indications for open reduction include open fractures or severely displaced fractures in adolescents with minimal growth remaining.  Acceptable limits of reduction in adolescent proximal humeral fractures include bayonet apposition and angulation of less than 35°.  Common blocks to reduction in adolescents include the biceps tendon and periosteum.  For this fracture, use of a shoulder sling without reduction will lead to healing and an excellent result as the proximal humerus remodels.

 

REFERENCES: Kohler R, Trillaud JM: Fracture and fracture separation of the proximal humerus in children: Report of 136 cases.  J Pediatr Orthop 1983;3:326-332.

Beaty JH: Fractures of the proximal humerus and shaft in children.  Instr Course Lect 1992;41:369-372.

Dobbs MB, Luhmann SL, Gordon JE, et al: Severely displaced proximal humeral epiphyseal fractures.  J Pediatr Orthop 2003;23:208-215.

Beringer DC, Weiner DS, Noble JS, et al: Severely displaced proximal humeral epiphyseal fractures: A follow-up study.  J Pediatr Orthop 1998;18:31-37.

Wang P Jr, Koval KJ, Lehman W, et al: Salter-Harris type III fracture-dislocation of the proximal humerus.  J Pediatr Orthop B 1997;6:219-222.

 

67.    What is the most common complication requiring reoperation after dorsal plating for a distal radius fracture?

 

1-         Extensor tenosynovitis

2-         Extensor tendon rupture

3-         Flexor pollicus longus tendon rupture

4-         Loss of reduction

5-         Dorsal impingement

 

PREFERRED RESPONSE: 1

 

DISCUSSION: The most common complication of dorsal plating is extensor tenosynovitis, which often causes pain and is a frequent reason for hardware removal.  Other less frequent complications include loss of reduction and extensor tendon ruptures, with flexor tendon ruptures occuring to an even lesser degree.

 

REFERENCES: Rozental TD, Beredjiklian PK, Bozentka DJ: Functional outcome and complications following two types of dorsal plating for unstable fractures of the distal part of the radius.  J Bone Joint Surg Am 2003;85:1956-1960.

Kambouroglou GK, Axelrod TS: Complications of the AO/ASIF titanium distal radius plate system (pi plate) in internal fixation of the distal radius: A brief report.  J Hand Surg Am 1998;23:737-741.

 

68.    Figures 36a and 36b show the radiographs of a 48-year-old woman who smokes cigarettes and sustained a segmental femoral shaft fracture in a motor vehicle accident 9 months ago.  Initial management consisted of stabilization with a reamed statically locked intramedullary nail.  She now reports lower leg pain that increases with activity.  In addition to advising the patient to quit smoking, management should include

 

1-         ultrasonic stimulation for 3 months.

2-         removal of the nail and plate fixation.

3-         continued observation.

4-         removal of the distal locking screws to dynamize the nail.

5-         exchange reamed nailing with bone graft.

 

PREFERRED RESPONSE: 5

 

DISCUSSION: The patient has an oligotrophic nonunion of the distal femoral fracture.  Although the proximal fracture appears incompletely united, it was stable at exchange nailing.  The treatment of choice is exchange reamed nailing to at least 2 mm above the nail in place.  Bone grafting is debatable.  Recent studies have shown a 70% to 75% success rate with exchange nailing only, so in nonhypertrophic nonunions, bone grafting can be considered.  Nonsurgical management consisting of observation or external stimulation runs the risk of implant failure.  Plate fixation is acceptable but is considered a second choice because of the need to consider stabilization of the proximal fracture until union is achieved.  Also, plate fixation definitely requires bone grafting.

 

REFERENCES: Webb LX, Winquist RA, Hansen ST: Intramedullary nailing and reaming for delayed union or nonunion of the femoral shaft: A report of 105 consecutive cases.  Clin Orthop 1986;212:133-141.

Weresh MJ, Hakanson R, Stover MD, et al: Failure of exchange reamed intramedullary nailing for ununited femoral shaft fractures.  J Orthop Trauma 2000;14:335-338.

Hak DG, Lee SS, Goulet JA: Success of exchange reamed intramedullary nailing for femoral shaft nonunion or delayed union.  J Orthop Trauma 2000;14:178-182.

 

69.    A 5-year-old boy has a deformity of his right arm after falling from a jungle gym. 
A radiograph is shown in Figure 37.  Management should consist of

 

1-         closed reduction of the ulna and transcapitellar pinning of the radial head.

2-         closed reduction of the ulna and radial head dislocation.

3-         closed reduction of the ulna and annular ligament repair.

4-         open reduction of the radius and plating of the ulna.

5-         open reduction of the ulna and immobilization in an extension cast.

 

PREFERRED RESPONSE: 2

 

DISCUSSION: Monteggia fractures in children must be recognized.  Early appropriate treatment is much easier than delayed reconstruction for a missed radial head dislocation.  In younger children, attempts should be made to reduce the ulna fracture and radial head dislocation with traction and manual manipulation.  Anterior Monteggia fractures are the most common, and in this variety the radius is much better stabilized in elbow flexion.  Posterior Monteggia fractures are less common and may be managed in elbow extension.  Closed reduction is much more successful in younger children; ulnar fixation with a rod or plate may be needed in older patients with unstable fractures.  Annular ligament repair is rarely needed in the acute fracture.

 

REFERENCES: Wilkins KE: Changes in the management of Monteggia fractures.  J Pediatr Orthop 2002;22:548-554.

Kay RM, Skaggs DL: The pediatric Monteggia fracture.  Am J Orthop 1998;27:606-609.

Ring D, Jupiter JB, Waters PM: Monteggia fractures in children and adults.  J Am Acad Orthop Surg 1998;6:215-224.

 

70.    What is the most important factor in determining recovery after surgical repair of a complete laceration of a nerve at the wrist?

 

1-         Timing of repair

2-         Technique of repair

3-         Patient’s age

4-         Use of a fibrin tissue sealant

5-         Use of a nerve conduit

 

PREFERRED RESPONSE: 3

 

DISCUSSION: All other factors being equal, a patient’s age is the most important factor in determining outcome after peripheral nerve injury.  Repair of a nerve laceration within the first 2 weeks is generally considered appropriate.  Fascicular repair may be of benefit in larger proximal nerves to reapproximate appropriate nerve bundles; distally perineural or epineural repair is sufficient.  Use of a fibrin tissue sealant for nerve repair does not result in improved outcomes over suture repair.  Nerve conduits have shown promise in digital nerves but do not have proven benefit in larger caliber nerves.

 

REFERENCES: Sunderland S: Nerve Injuries and Their Repair: A Critical Appraisal.  New York, NY, Churchill Livingstone, 1991.

Wilgis ES, Brushart TM: Nerve repair and grafting, in Green DP, Hotchkiss RN (eds): Operative Hand Surgery, ed 3.  New York, NY, Churchill Livingstone, 1993, p 1325.

Narakas A: The use of fibrin glue in repair of peripheral nerves.  Orthop Clin North Am 1988;19:187-199.

Weber RA, Breidenbach WC, Brown RE, et al: A randomized prospective study of
polyglycolic acid conduits for digital nerve reconstruction in humans.  Plast Reconstr Surg 2000;106:1036-1045.

 

71.    A 39-year-old woman fell onto her flexed elbow and sustained a comminuted displaced radial head and neck fracture.  Radiographs confirm concentric reduction of the ulnohumeral joint.  Examination reveals pain with compression of the radius and ulna at the wrist.  What is the best treatment for the radial head fracture?

 

1-         Long arm cast for 2 weeks, followed by range of motion

2-         Early range of motion

3-         Metallic radial head arthroplasty

4-         Silastic radial head arthroplasty

5-         Excision of the radial head

 

PREFERRED RESPONSE: 3

 

DISCUSSION: Patients with comminuted radial neck and head fractures and associated wrist pain have a significant injury to the elbow and forearm.  Nonsurgical management is an option, but initial casting will result in stiffness and early range of motion is likely to be unsuccessful secondary to pain.  Surgical treatment with open reduction and internal fixation, although possible, is technically demanding and results are unpredictable with comminuted fractures.  Excision alone in the face of wrist pain may lead to radial shortening.  The treatment of choice is excision and metallic radial head arthroplasty.  Silastic implants have been associated with synovitis and wear debris.

 

REFERENCES: Furry KL, Clinkscales CM: Comminuted fractures of the radial head: Arthroplasty versus internal fixation.  Clin Orthop 1998;353:40-52.

Ring D, Quintero J, Jupiter JB: Open reduction and internal fixation of fractures of the radial head.  J Bone Joint Surg Am 2002;84:1811-1815.

 

72.    A 25-year-old laborer sustains a transverse fracture of the proximal 25% of the scaphoid.  CT reconstructions reveal a 1-mm fracture gap.  What is the most appropriate treatment?

 

1-         Above-elbow thumb spica cast

2-         Short arm thumb spica cast

3-         Scaphotrapezial-trapezoidal (STT) fusion

4-         Excision of the proximal pole

5-         Internal fixation of the fracture with a compression screw

 

PREFERRED RESPONSE: 5

 

DISCUSSION: A higher risk of nonunion and the need for prolonged immobilization is seen after nonsurgical management of proximal pole fractures of the scaphoid.  Because of the relatively poor blood supply of the proximal pole, surgical treatment with a compression screw is advocated for fractures of the proximal third of the scaphoid.

 

REFERENCES: Clay NR, Dias JJ, Costigan PS, et al: Need the thumb be immobilized in scaphoid fractures?  A randomised prospective trial.  J Bone Joint Surg Br 1991;73:828-832.

Ring D, Jupiter JB, Herndon JH: Acute fractures of the scaphoid.  J Am Acad Orthop Surg 2000;8:225-231.

 

73.    A 34-year-old man sustained a tibial fracture in a motorcycle accident.  What perioperative variable is associated with the greatest relative risk for reoperation to achieve bone union?

 

1-         Gender

2-         Delay in initial surgical treatment

3-         Use of nonsteroidal anti-inflammatory drugs

4-         Smoking

5-         Cortical contact of less than or equal to 50%

 

PREFERRED RESPONSE: 5

 

DISCUSSION: In a recent analysis of 200 patients with tibial fractures, Bhandari and associates attempted to identify variables that were predictive of reoperation.  The variables in the study were type of injury (fracture pattern), degree of open injury, mechanism of injury, cortical bone contact, postoperative complications, polytrauma, anti-inflammatory drug use, nail insertion technique (reamed versus nonreamed), smoking history, alcohol use, diabetes mellitus, peripheral vascular disease, age, disability status pre-injury, gender, surgeon, time to surgery, steroid use, phenytoin use, antibiotic use, anticoagulant use, and type of fixation used.  Three variables were statistically significant predictors of reoperation to achieve bone union in the first postinjury year: transverse fracture pattern, open fracture, and cortical contact of 50% or less.  Using these three variables, four reoperation risk groups were identified based on the number of these three variables present: 0, 1, 2, or 3.  The risk for reoperation was 0%, 18%, 47%, and 94%, respectively.  The authors concluded that these statistics can provide prognostic information to patients and help identify those high-risk patients where early intervention to achieve union is indicated.  In addition, the data highlights the significance of achieving cortical contact at the time of initial fixation.

 

REFERENCE: Bhandari M, Tornetta P III, Sprague S, et al: Predictors of reoperation following operative management of fractures of the tibial shaft.  J Orthop Trauma 2003;17:353-361.

 

74.    Figure 38a shows the radiograph of a 12-year-old boy who underwent a reamed intramedullary nailing for a closed femoral shaft fracture.  One year after rod removal, he reports groin pain.  A current radiograph is shown in Figure 38b.  The findings are most likely the result of

 

1-         a torn ligamentum teres.

2-         damage to the femoral neck.

3-         damage to the lateral ascending vessels of the femoral neck.

4-         unrecognized Perthes’ disease.

5-         growth arrest of the proximal physis.

 

PREFERRED RESPONSE: 3

 

DISCUSSION: Osteonecrosis of the femoral head is a known complication from the use of rigid intramedullary nails for femoral fractures in adolescents.  When the nails are placed through the piriformis fossa, the lateral ascending vessels of the femoral neck may be injured, resulting in osteonecrosis of the femoral head in 1% to 2% of patients.  Rigid reamed nails placed into the piriformis fossa are contraindicated in children with open growth plates because the physis is a barrier to blood supply and the ligamentum teres does not provide sufficient vascularity.  Alternative fixation methods for femoral fractures in adolescents include external fixation and open reduction and internal fixation.  Nailing through the tip of the trochanter may decrease the incidence of this serious complication.

 

REFERENCES: Letts M, Jarvis J, Lawton L, et al: Complications of rigid intramedullary rodding of femoral shaft fractures in children.  J Trauma 2002;52:504-516.

Stans AA, Morrissy RT, Renwick SE: Femoral shaft fracture treatment in patients age 6 to 16 years.  J Pediatr Orthop 1999;19:222-228.

Buckley SL: Current trends in the treatment of femoral shaft fractures in children and adolescents.  Clin Orthop 1997;338:60-73.

Beaty JH, Austin SM, Warner WC, et al: Interlocking intramedullary nailing of femoral-shaft fractures in adolescents: Preliminary results and complications.  J Pediatr Orthop
1994;14:178-183.

 

75.    A 36-year-old woman has neck pain in the upper cervical region and occipital discomfort after being involved in a motor vehicle accident.  Examination reveals no forehead or scalp lacerations.  The neurologic examination is normal.  A CT scan shows no evidence of bony injury.  Figures 39a and 39b show a lateral radiograph and an MRI scan.  Management should consist of

 

1-         a hard cervical collar for 6 weeks.

2-         skeletal traction for 6 weeks, followed by halo vest immobilization for 6 weeks.

3-         halo vest immobilization for 3 months.

4-         posterior cervical C1-2 wiring with arthrodesis.

5-         anterior C2-3 diskectomy, fusion, and plating.

 

PREFERRED RESPONSE: 4

 

DISCUSSION: The lateral radiograph shows 8 mm of atlantoaxial translation.  In the absence of a bony injury, this represents rupture of the transverse atlantal ligament. The MRI scan reveals soft-tissue swelling posterior to the odontoid and a high intensity zone in the atlanto-dens interval consistent with acute injury. These injuries require arthrodesis because nonsurgical measures will not provide stability.  Techniques for C1-2 fusion include Gallie, Brooks, or triple wiring.  Transarticular screw fixation across the C1-2 articulation provides the most rigid means of fixation and the highest arthrodesis rates but is technically demanding.  Anterior C2-3 arthrodesis will not address the level of instability.  The normal atlanto-dens interval is 3 mm in an adult and 4 mm in a child.

 

REFERENCES: Kurz LT: Transverse atlantal ligament insufficiency, in Clark CR (ed):
The Cervical Spine.  Philadelphia, PA, Lippincott-Raven, 1998, pp 401-407.

Fielding JW, Cochran GB, Lansing JF III, et al: Tears of the transverse ligament of the atlas:
A clinical and biomechanical study.  J Bone Joint Surg Am 1974;56:1683-1691.

 

76.    A 30-year-old woman sustains a transverse amputation of the distal phalanx of the index finger, leaving exposed bone.  What is the most appropriate management of the soft-tissue defect?

 

1-         Dressing changes and healing by secondary intention

2-         Split-thickness skin grafting

3-         V-Y advancement flap

4-         Moberg (volar advancement flap)

5-         First dorsal metacarpal artery-island pedicled flap

 

PREFERRED RESPONSE: 3

 

DISCUSSION: V-Y advancement flaps are ideal for fingertip amputations that are transverse or dorsal oblique in nature.  Healing by secondary intention is contraindicated with exposed bone.  Shortening of exposed bone to allow primary skin closure is a possible alternative, as long as significant shortening of the index finger is avoided.  A Moberg flap is useful only for distal amputations of the thumb.  The first dorsal metacarpal artery-island pedicled flap uses tissue from the dorsum of the proximal index finger, and is typically used to resurface defects of
the thumb.

 

REFERENCES: Fassler PR: Fingertip injuries: Evaluation and treatment.  J Am Acad Orthop Surg 1996;4:84-92.

Atasoy E, Ioakimidis E, Kasdan ML, et al: Reconstruction of the amputated fingertip with a triangular volar flap: A new surgical procedure.  J Bone Joint Surg Am 1970;52:921-926.

 

77.    What is the best approach to reduce and stabilize a displaced volar lunate facet fracture of the wrist?

 

1-         Extended carpal tunnel incision

2-         Dorsal midline

3-         Interval between the flexor carpi radialis and radial artery

4-         Just radial to the ulnar artery

5-         Dorsal approach between the ulna and radius over the sigmoid notch

 

PREFERRED RESPONSE: 1

 

DISCUSSION: A volar lunate fragment of a distal radial fracture is considered a critical component to overall joint stability and function.  Obtaining a reduction is difficult through a standard volar approach to the radius between the flexor carpi radialis and radial artery.  Visualization and reduction of the ulnar volar facet is not possible from this approach.   An extended carpal tunnel incision provides access to the entire articular surface, except for the distal radial styloid component. 

 

REFERENCES: Hanel DP, Jones MD, Trumble TE: Wrist fractures.  Orthop Clin North Am 2002;33:35-57.

Trumble TE, Culp RW, Hanel DP, et al: Intra-articular fractures of the distal aspect of the radius.  Instr Course Lect 1999;48:465-480.

 

78.    A 17-year-old man sustained a 5-mm laceration on the lateral aspect of the hindfoot while working on a farm.  Examination in the emergency department revealed no fractures.  Twenty-four hours later, he returns to the emergency department with increasing foot pain.  Thin brown drainage is seen emanating from the wound.  He has a temperature of 102.0° F (38.9° C), a pulse rate of 120, and a blood pressure of 80/40 mm Hg.  Examination of the foot reveals diffuse swelling, ecchymosis, tenderness, and crepitus with palpation.  Current radiographs are shown in Figures 40a and 40b.  Management should now consist of

 

1-         intravenous antibiotics.

2-         hyperbaric oxygen therapy and intravenous antibiotics.

3-         surgical debridement, primary wound closure, and intravenous antibiotics.

4-         surgical debridement, closure of the wound over drains, and intravenous antibiotics.

5-         surgical debridement, leaving the wound open, and intravenous antibiotics.

 

PREFERRED RESPONSE: 5

 

DISCUSSION: The mechanism and environment in which the injury occurred, the clinical picture, and the radiographic findings of gas in the tissues suggest an anaerobic Gram-positive bacterial infection.  This can be a life- and limb-threatening infection.  Treatment should consist of wide debridement of all devitalized tissue, and intravenous antibiotics should be started.  Wounds should be left open to allow bacterial effluent and increase oxygen tension in the wound.  Hyperbaric oxygen may be used as an adjuvant but is no substitute for debridement.

 

REFERENCES: Pellegrini VD, Reid JS, Evarts CM: Complications, in Rockwood CA, Green DP, Bucholz RW, et al (eds): Rockwood and Green’s Fractures in Adults, ed 4.  Philadelphia, PA, Lippincott-Raven, 1996, vol 1, pp 458-463.

Ayers DC, Murray DC: Complications of the treatment of fractures and dislocations: General considerations, in Epps Jr CH (ed): Complications in Orthopedic Surgery, ed 4.  Philadelphia, PA, JB Lippincott, 1994, pp 3-48.

 

79.    A healthy, active, independent 74-year-old woman fell and sustained the elbow injury shown in Figures 41a and 41b.  Management should consist of

 

1-         a sling and early elbow range-of-motion exercises.

2-         a long arm cast for 6 weeks.

3-         open reduction and internal fixation.

4-         total elbow arthroplasty.

5-         elbow arthrodesis.

 

PREFERRED RESPONSE: 4

 

DISCUSSION: Open reduction and internal fixation of distal humeral fractures in elderly patients often fails.  These fractures characteristically have a very small distal segment and poor bone quality, resulting in failure of fixation and nonunion.  Nonunion is often painful and functionally debilitating.  Total elbow arthroplasty provides good results when used for distal humeral fractures in elderly patients with osteopenic bone and fracture patterns thought to be irreconstructable.  Long arm casting may result in union, but the resulting stiffness is unacceptable for an active patient.  Elbow arthrodesis has few indications.  A sling and range-of-motion exercises will often result in a painful and debilitating nonunion at the fracture site. 

 

REFERENCES: Frankle MA, Herscovici D Jr, DiPasquale TG, et al:  A comparison of open reduction and internal fixation and primary total elbow arthroplasty in the treatment of intra-articular distal humerus fractures in women older than 65.  J Orthop Trauma 2003;17:473-480.

Cobb TK, Morrey BF: Total elbow arthroplasty as primary treatment for distal humerus fractures in elderly patients.  J Bone Joint Surg Am 1997;79:826-832.

Obremskey WT, Bhandari M, Dirschl DR, et al: Internal fixation versus arthroplasty of comminuted fractures of the distal humerus.  J Orthop Trauma 2003;17:463-465.

 

80.    A 35-year-old man sustains a closed Monteggia fracture.  Examination reveals that sensation, vascular status, and finger flexion are normal.  When he extends his wrist, it deviates radially, and he is unable to extend his fingers or thumb.  After reduction
of the fracture, what is the next step in treatment for the extensor deficits of the thumb and fingers?

 

1-         Exploration of the radial nerve

2-         Exploration of the median nerve

3-         Nerve conduction velocity studies

4-         Tendon transfers after the fracture is stabilized

5-         Observation

 

PREFERRED RESPONSE: 5

 

DISCUSSION: The posterior interosseous nerve is located adjacent to the radial neck, placing it at risk for a traction injury with a dislocation of the proximal radius. The typical neurapraxia that results can be expected to resolve with observation within the first 6 to 12 weeks. If recovery is not clinically evident by 3 months, neurophysiologic studies are indicated.

 

REFERENCES: Jessing P: Monteggia lesions and their complicating nerve damage.  Acta Orthop Scand 1975;46:601-609.

Stein F, Grabias SL, Deffer PA: Nerve injuries complicating Monteggia lesions.  J Bone Joint Surg Am 1971;53:1432-1436.

 

81.    A 25-year-old man is brought to the emergency department following a motor vehicle accident.  Extrication time was 2 hours, and in the field he had a systolic blood pressure by palpation of 90 mm Hg.  Intravenous therapy was started, and on arrival in the emergency department he has a systolic blood pressure of 90 mm Hg with a pulse rate of 130.  Examination reveals a flail chest and a femoral diaphyseal fracture.  Ultrasound of the abdomen is positive.  The trauma surgeons take him to the operating room for an exploratory laparotomy.  At the conclusion of the procedure, he has a systolic pressure of 100 mm Hg with a pulse rate of 110.  Oxygen saturation is 90% on 100% oxygen, and he has a temperature of 95.0° F (35° C).  What is the recommended treatment of the femoral fracture at this time?

 

1-         Reamed intramedullary nail

2-         Unreamed intramedullary nail

3-         Percutaneous plate fixation

4-         Skeletal traction

5-         External fixation

 

PREFERRED RESPONSE: 5

 

DISCUSSION: This is a “borderline trauma” patient where serious consideration for damage control orthopaedic surgery is required.  His prolonged hypotension, abdominal injury, and chest injury put him at higher risk for serious postinjury complications.  Further surgery, such as definitive fracture fixation, adds metabolic load and injury to his system.  It is prudent to consider femoral fracture stabilization with an external fixator until he is physiologically recovered as evidenced by a normal base excess and/or lactate acid levels, as well as all other parameters of resuscitation.  A borderline patient has been described as polytrauma with an ISS > 20 and thoracic trauma (AIS > 2); polytrauma and abdominal/pelvic trauma (Moore > 3) and hemodynamic shock (initial BP < 90 mm Hg); ISS > 40; bilateral lung contusions on radiographs; initial mean pulmonary arterial pressure > 24 mm Hg; pulmonary artery pressure increase during intramedullary nailing > 6 mm Hg.  Factors that worsen the situation following surgery include multiple long bones and truncal injury (AIS > 2), estimated surgery time of more than 6 hours, arterial injury and hemodynamic instability, and exaggerated inflammatory response (eg, Il-6 > 800 pg/mL).  It is incumbent on the orthopaedic surgeon who is a member of the trauma team to make sure that he or she is aware of these factors and guides the team to the best patient care.

 

REFERENCES: Pape HC, Hildebrand F, Pertschy S, et al: Changes in the management of femoral shaft fractures in polytrauma patients: From early total care to damage control orthopaedic surgery.  J Trauma 2002;53:452-461.

Bosse M, Kellam JF: Orthopaedic decision making in the multiple trauma patient, in Browner BD (ed): Skeletal Trauma, ed 3.  Philadelphia, PA, WB Saunders, 2003, pp 133-146.

 

82.    A 10-year-old girl has a right elbow deformity that is the result of trauma 5 years ago.  She has no pain despite the arm deformity.  The radiographs in Figures 42a and 42b show complete healing.  This radiographic appearance demonstrates what complication?

 

1-         Growth arrest of the medial trochlear physis

2-         Varus malunion of a supracondylar humeral fracture

3-         Valgus malunion of a lateral condylar fracture

4-         Posterior and lateral dislocation of the radial head

5-         Osteonecrosis of the capitellum

 

PREFERRED RESPONSE: 2

 

DISCUSSION: Cubitus varus is a common complication of displaced supracondylar humeral fractures that are treated with closed reduction and cast immobilization.  Treatment with closed reduction and percutaneous pinning decreases the incidence of this complication.  Cubitus varus also can occur in minimally displaced fractures when unrecognized collapse of the medial column of the distal humerus is not corrected with manipulation.  This can be detected on physical examination of the carrying angle or on radiographs measuring Baumann’s angle, both in comparison to the opposite side.  Cubitus varus may result in unacceptable cosmesis and may predispose the patient to fractures of the lateral condyle.  The lateral radiograph demonstrates the crescent sign from overlap of the distal humerus with the olecranon seen in patients with cubitus varus.  Patients with growth arrest to the medial trochlear physis would have atrophy of the trochlea on radiographs.

 

REFERENCES: Flynn JM, Sarwark JF, Waters PM, et al: The surgical management of pediatric fractures of the upper extremity. Instr Course Lect 2003;52:635-45.

Papandrea R, Waters PM: Posttraumatic reconstruction of the elbow in the pediatric patient.  Clin Orthop 2000;370:115-126.

Lins RE, Simovitch RW, Waters PM: Pediatric elbow trauma.  Orthop Clin North Am 1999;30:119-132.

 

83.    A 64-year-old woman has left wrist pain and deformity after falling on her hand.  Examination shows intact skin and no neurologic or vascular injuries.  Radiographs are shown in Figures 43a and 43b.  What is the most appropriate management for the injury?

 

1-         Closed reduction and above-elbow cast immobilization in supination

2-         Closed reduction and joint spanning external fixation

3-         Closed reduction and percutaneous pinning followed by cast immobilization

4-         Open reduction through a dorsal approach and fixation with an angular
stable plate

5-         Open reduction through a volar approach and stabilization with a buttress plate

 

PREFERRED RESPONSE: 5

 

DISCUSSION: The patient has a volar displaced two-part intra-articular distal radial fracture-dislocation of the wrist.  Although a closed reduction is usually easily obtained, it is very difficult to maintain the reduction without internal fixation.  The approach is determined by the direction of the dislocation, in this case volar.  Stabilization with a buttress plate neutralizes the axial loading forces on the fractured fragment.  A dorsal placed angular stable plate will not provide this buttress effect and will make the reduction difficult.

 

REFERENCE: Cohen M, McMurtry RY, Jupiter JB: Fractures and dislocations of the carpus, in Browner BD (ed): Skeletal Trauma, ed 3.  Philadelphia, PA, WB Saunders, 2003, pp 1328-1335.

 

84.    A 26-year-old man was thrown from a car and sustained the injury seen in Figures 44a and 44b.  Nonsurgical management of this injury is recommended.  Which of the following factors increases the risk of nonunion?

 

1-         Male gender

2-         Diaphyseal location

3-         Comminuted displaced fracture

4-         Young age

5-         Associated injuries

 

PREFERRED RESPONSE: 3

 

DISCUSSION: The patient has a displaced comminuted clavicle middle one third fracture from a high-energy mechanism.  Recent literature on high-energy clavicular fractures suggests a higher rate of nonunion than previously reported.  A nonunion rate of 30% has been reported by Hill and associates when the fracture fragments are displaced more than 1.5 cm.  In addition, several patients had neurologic symptoms related to the injury.  Robinson and associates reported an increased risk of nonunion in women, elderly patients, comminuted fractures, and injuries with a lack of cortical contact.

 

REFERENCES: Hill JM, McGuire MH, Crosby LA: Closed treatment of displaced middle-third fractures of the clavicle gives poor results.  J Bone Joint Surg Br 1997;79:537-539.

Wick M, Muller EJ, Kollig E: Midshaft fractures of the clavicle with a shortening of more than
2 cm predispose to nonunion.  Arch Orthop Trauma Surg 2001;121:207-211.  

Robinson CM, Court-Brown CM, McQueen MM, et al: Estimating the risk of nonunion following nonoperative treatment of a clavicular fracture.  J Bone Joint Surg Am
2004;86:1359-1365.

 

85.    A 13-year-old girl was riding on an all-terrain vehicle when the driver struck a tree.  She sustained the injury shown in Figures 45a through 45d.  This injury is best described as what type of acetabular fracture pattern?

 

1-         T-type

2-         Anterior column

3-         Both-column

4-         Anterior column posterior hemitransverse

5-         Posterior column

 

PREFERRED RESPONSE: 3

 

DISCUSSION: The fracture is a both-column fracture in the Judet/Letournel classification and a C3 in the AO classification.  There is extension into the sacroiliac joint along the pelvic brim and comminution along the posterior column above the sciatic notch.  Both the anterior and posterior columns are separately broken and displaced.  However, the defining feature of a both-column pattern, as seen in this patient, is that all articular fragments are on fracture fragments and no joint surface is left intact to the axial skeleton above.  The use of three-dimensional images makes it easier to view the location of the fracture fragments and the amount and direction
of displacement.

 

REFERENCES: Helfet DL, Beck M, Gautier E, et al: Surgical techniques for acetabular fractures, in Tile M, Helfet DL, Kellam JF (eds): Fractures of the Pelvis and Acetabulum.  Philadelphia, PA, Lippincott Williams & Wilkins, 2003, pp 533-603.

Tile M: Describing the injury: Classification of acetabular fractures, in Tile M, Helfet DL, Kellam JF (eds): Fractures of the Pelvis and Acetabulum, ed 3.  Philadelphia, PA, Lippincott Williams & Wilkins, 2003, pp 427-475.

Brandser E, Marsh JL: Acetabular fractures: Easier classification with a systematic approach.  Am J Roentgenol 1998;171:1217-1228.

86.    A woman injures the metacarpophalangeal (MCP) joint of her thumb while skiing.  Examination reveals tenderness along the ulnar aspect of the MCP joint.  Radially directed stress of the joint in full extension produces 5° of angulation.  When the MCP joint is flexed 30°, a radially directed stress produces 45° of angulation.  Radiographs are otherwise normal.  Management should consist of

 

1-         a thumb spica cast and reassessment in 3 weeks.

2-         a thumb spica cast and reassessment in 6 weeks.

3-         repair of the ulnar collateral ligament of the MCP joint.

4-         adductor pollicis advancement on the proximal phalanx.

5-         repair of the MCP joint dorsal capsule.

 

PREFERRED RESPONSE: 1

 

DISCUSSION: Injuries to the ulnar collateral ligament of the MCP joint of the thumb commonly occur in recreational skiers.  Historically, this injury has been referred to as “gamekeeper’s thumb.”  The ligament consists of the proper collateral ligament and the more volar accessory collateral ligament.  In extension, the accessory ligament is taut, and in flexion, the proper ligament is taut.  For a complete tear of the ligament complex to occur, there must be laxity in full extension.  Incomplete tears respond well to thumb spica splinting or casting for 2 to 3 weeks and gradual resumption of range of motion.  Prolonged immobilization of incomplete injuries leads to higher rates of MCP joint stiffness.

 

REFERENCES: Stener B: Displacement of the ruptured ulnar collateral ligament of the metacarpo-phalangeal joint of the thumb: A clinical and anatomical study.  J Bone Joint Surg Br 1971;44:869.

Heyman P: Injuries to the ulnar collateral ligament of the thumb metacarpophalangeal joint. 
J Am Acad Orthop Surg 1997;5:224-229.

 

87.    A 37-year-old laborer falls 12 feet and sustains a comminuted tibial plafond
fracture.  Three years after treatment using standard techniques, what will be
the most likely outcome?

 

1-         Need for ankle fusion or arthroplasty

2-         Return to normal function

3-         Ankle stiffness without pain

4-         Severe constant pain and inability to work

5-         Adversely affected general health status and posttraumatic arthritis

 

PREFERRED RESPONSE: 5

 

DISCUSSION: Two recent studies by Pollak and associates and Marsh and associates have focused on function after high-energy tibial plafond fractures. Findings are unfavorable even when anatomic reduction is performed in the best centers and patients are provided excellent rehabilitation. Function improves up to 2 years after injury, but even basic walking skills remain adversely affected.  Virtually all patients have long-term adverse general health effects compared to their gender and age-matched peers.  Posttraumatic degenerative arthritis is present in most ankles.  Patients should be told early about the long-term prognosis, and early vocational/psychological counseling should be given.  Despite these adverse outcomes, only a minority of patients require fusion or arthroplasty.

 

REFERENCES: Pollak AN, McCarthy ML, Bess RS, et al: Outcomes after treatment of high-energy tibial plafond fractures.  J Bone Joint Surg Am 2003;85:1893-1900.

Marsh JL, Weigel DP, Dirschl DR: Tibial plafond fractures: How do these ankles function over time?  J Bone Joint Surg Am 2003;85:287-295.

 

88.    A 45-year-old woman sustains an injury to her lower leg.  Examination reveals that there is a deformity with no neurologic or vascular problems.  The skin is intact.  Radiographs are shown in Figures 46a and 46b.  Which of the following factors would make closed management the least appropriate choice for this injury?

 

1-         Spiral fracture pattern

2-         Low-energy mechanism

3-         Amount of shortening

4-         Fracture of the fibula at a different level

5-         Ipsilateral femoral fracture

 

PREFERRED RESPONSE: 5

 

DISCUSSION: All the factors listed, with the exception of an ipsilateral femoral fracture, are representative of a low-energy stable tibial shaft fracture that will do well with closed reduction and immobilization in a long leg cast, followed by weight bearing as tolerated and then a functional brace or patellar tendon bearing cast until union is achieved.  Shortening will not increase from that seen on these initial radiographs.  The spiral fracture provides a broad surface for healing, and the fibular fracture at another level indicates a stable soft-tissue envelope which, with the immobilization device, will stabilize the fracture reduction.  An ipsilateral femoral fracture is a strong indication to surgically stabilize both fractures.

 

REFERENCES: Trafton PG: Tibial shaft fractures, in Browner BD (ed): Skeletal Trauma, ed 3.  Philadelphia, PA, WB Saunders, 2003, pp 2153-2169.

Martinez A, Sarmiento A, Latta LL: Closed fractures of the proximal tibia treated with a functional brace.  Clin Orthop 2003;417:293-302.

 

89.    Which of the following medications may have a negative effect on bone healing following fracture?

 

1-         COX-1 nonsteroidal anti-inflammatory drugs

2-         Phenytoin

3-         Temazepam

4-         Serotonin reuptake blockers (SSRIs)

5-         Amitriptyline

 

PREFERRED RESPONSE: 1

 

DISCUSSION: Nonsteroidal anti-inflammatory drugs that are COX-1 primary inhibitors have been shown in animal studies to delay or inhibit fracture healing.  COX-2 inhibitors also delay healing but to a lesser extent than COX-1 inhibitors.  The other medications listed do not alter fracture callus formation. 

 

REFERENCES: Gerstenfeld LC, Thiede M, Seibert K, et al: Differential inhibition of fracture healing by non-selective and cyclooxygenase-2 selective non-steroidal anti-inflammatory drugs.  J Orthop Res 2003;21:670-675.

Harder AT, An YH: The mechanisms of the inhibitory effects of nonsteroidal anti-inflammatory drugs on bone healing: A concise review.  J Clin Pharmacol 2003;43:807-815.

 

90.    A 16-year-old boy has abdominal and back pain after being involved in a high-velocity head-on motor vehicle accident.  He was restrained in the rear of the automobile by a lap belt only.  A radiograph and CT scan are shown in Figure 47.  The patient has no other injuries.  Optimal management should include

 

1-         bed rest for 6 weeks.

2-         open reduction and internal fixation with spinous process wiring.

3-         cast immobilization in hyperextension for 6 weeks, followed by a thoracolumbosacral orthosis.

4-         anterior corpectomy, tricortical autograft, and fixation with a plate and screws.

5-         posterior fixation with a pedicle screw construct.

 

PREFERRED RESPONSE: 3

 

DISCUSSION: Pediatric bony Chance fractures occur following severe flexion injuries as seen after motor vehicle accidents with lap belt restraints.  There is a high rate of associated intra-abdominal injuries.  In the absence of associated injuries, these fractures are best treated with immobilization.  Bed rest is not necessary.  Surgical fixation usually is not needed.  Surgical stabilization and two-level fusion may be indicated in select individuals with progressive kyphosis of more than 25° or other conditions that preclude cast or brace immobilization.

 

REFERENCES: Greenwald TA, Mann DC: Pediatric seatbelt injuries: Diagnosis and treatment of lumbar flexion-distraction injuries.  Paraplegia 1994;32:743-751.

Glassman SD, Johnson JR, Holt RT: Seatbelt injuries in children.  J Trauma 1992;33:882-886.

Raney EM, Bennett JT: Pediatric Chance fracture.  Spine 1992;17:1522-1524.

 

91.    What inflammatory mediator has been most closely associated with the magnitude of the systemic inflammatory response to trauma and with the development of multiple organ dysfunction syndrome (MODS)?

 

1-         IL-1

2-         IL-6

3-         TNF

4-         TGF-ß

5-         CRP

 

PREFERRED RESPONSE: 2

 

DISCUSSION: Multiple cytokines (inflammatory mediators) are released following trauma, and their levels can be measured in serum.  Persistent elevated levels of IL-6 (> 800 pg/mL) indicate an exaggerated systemic inflammatory response to trauma and have been associated with the development of MODS.  Recent work has shown that extensive surgical procedures should be avoided when IL-6 levels remain elevated to prevent the precipitation of MODS.  In the future, it is likely that this mediator and possibly others will be used to determine timing and techniques of future treatment.

 

REFERENCES: Patrick DA, Moore FA, Moore EE, et al: Jack A. Barney Resident Research Award winner: The inflammatory profile of interleukin-6, interleukin-8, and soluble intercellular adhesion molecule-1 in postinjury multiple organ failure.  Am J Surg 1996;172:425-429.

Pape HC, van Griesven M, Rice J, et al: Major secondary surgery in blunt trauma patients and perioperative cytokine liberation: Determination of the clinical relevance of biochemical markers.   J Trauma 2001;50:989-1000.

Giannoudis PV: When is the safest time to undertake secondary definitive fracture stabilization procedures in multiply injured patients who were initially managed using a strategy of primary temporary skeletal fixation.  J Trauma 2002;52:811-812.

 

92.    An 8-year-old girl sustained a displaced fracture at the base of the femoral neck in a motor vehicle accident.  Management should consist of

 

1-         closed reduction and spica cast immobilization.

2-         closed reduction and fixation of the femoral fracture with smooth pins across
the physis.

3-         open reduction and fixation with screws across the fracture and short of the growth plate, and a spica cast.

4-         skeletal traction in a 90 - 90 position.

5-         temporary traction until fixation with a specially ordered pediatric hip screw
is possible.

 

PREFERRED RESPONSE: 3

 

DISCUSSION: Pediatric intracapsular hip fractures are challenging because of the high rates of complications, including osteonecrosis and varus malunion.  These patients should be treated as emergencies.  Principles of treatment include anatomic reduction with internal fixation.  Screw fixation short of the physis is preferred and may need to be supplemented with spica cast immobilization.  Fixation may be achieved with smooth pins across the physis when little metaphyseal bone is available.  Fixation across the physis with threaded screws is acceptable only when the patient is close to skeletal maturity.  Pediatric hip screws are appropriate if immediately available.  Emergent open reduction, capsulotomy, or joint aspiration may decrease the rate of osteonecrosis.

 

REFERENCES: Cheng JC, Tang N: Decompression and stable internal fixation of femoral neck fractures in children can affect the outcome.  J Pediatr Orthop 1999;19:338-343.

Azouz EM, Karamitsos C, Reed MH, et al: Types and complications of femoral neck fractures in children.  Pediatr Radiol 1993;23:415-420.

Song KS, Kim YS, Sohn SW, et al: Arthrotomy and open reduction of the displaced fracture of the femoral neck in children.  J Pediatr Orthop B 2001;10:205-210.

Morsy HA: Complications of fracture of the neck of the femur in children: A long-term follow-up study.  Injury 2001;32:45-51.

 

93.    The plate seen in Figure 48a was applied to the fracture seen in Figure 48b, and is functioning in what capacity?

 

1-         Buttress

2-         Neutralization

3-         Tension band

4-         Compression

5-         Distraction

 

PREFERRED RESPONSE: 2

 

DISCUSSION: A Weber type B ankle fracture occurs with a supination external rotation mechanism of injury.  The fibula generally fails with a spiral fracture pattern.  The lag screws provide compression, and the plate acts to neutralize rotational and angular bending forces.  A buttress plate resists vertical shear forces.  A tension band is used over areas that may fail in tension, such as an olecranon fracture.  Compression is provided by the lag screws, and distraction is again resisted by the lag screws.  

 

REFERENCE: Mazzoca AD: Principles of internal fixation, in Browner BD, Jupiter JB, Levine AM, et al (eds): Skeletal Trauma, ed 2.  Philadelphia, PA, WB Saunders, 1998, pp 308-309.

 

94.    Which of the following findings is considered the strongest indication for surgical treatment of a mallet fracture of the distal phalanx?

 

1-         Fragment size of more than 20% of the articular surface

2-         Displacement of more than 2 mm

3-         Articular step-off of more than 2 mm

4-         Articular impaction

5-         Volar subluxation of the distal phalanx

 

PREFERRED RESPONSE: 5

 

DISCUSSION: The majority of mallet fractures can be treated nonsurgically with a distal interphalangeal joint extension splint.  Excellent results can be obtained in most patients with splinting alone.  The fragment size, amount of displacement, and degree of articular incongruity usually do not affect final outcome, as long as the joint is reduced.  Surgical fixation takes on several forms but is fraught with complications including skin/wound problems, loss of fixation, nonunion, and stiffness of the distal interphalangeal joint.  Volar subluxation of the distal phalanx remains the primary indication for surgical treatment.

 

REFERENCES: Green DP, Butler TE Jr: Fractures and dislocations in the hand, in Rockwood CA, Green DP, Bucholz RW, Heckman JD (eds): Rockwood and Green’s Fractures in Adults, ed 4.  Philadelphia, PA, Lippincott-Raven, 1996, pp 621-623.

Light TR (ed): Hand Surgery Update 2.  Rosemont, IL, American Academy of Orthopaedic Surgeons, 1999, pp 19-28.

 

95.    A 20-year-old woman sustained the closed injury shown in Figures 49a and 49b in a motor vehicle accident.  Examination reveals that this is an isolated injury; however, she has a complete radial nerve palsy.  Management should consist of

 

1-         splinting for 1 to 2 weeks, followed by a humeral fracture brace.

2-         intramedullary nailing.

3-         exploration of the radial nerve and intramedullary nailing.

4-         exploration of the radial nerve and a humeral fracture brace.

5-         exploration of the radial nerve and open reduction and internal fixation with plates and screws.

 

PREFERRED RESPONSE: 1

 

DISCUSSION: Lacerated radial nerves are associated with open humeral fractures.  All open humeral fractures with radial nerve palsy should be managed with radial nerve exploration and skeletal stabilization.  Closed humeral fractures with associated radial nerve palsy usually have an intact nerve with neurapraxia.  Most of these patients recover without surgical treatment.  If the patient has multiple injuries, skeletal stabilization may be indicated to improve mobilization.  For an isolated closed humeral fracture with a radial nerve palsy, the treatment of choice is splinting for 1 to 2 weeks, followed by a humeral fracture brace.

 

REFERENCES: Ring D, Chin K, Jupiter JB: Radial nerve palsy associated with high-energy humeral shaft fractures.  J Hand Surg Am 2004;29:144-147.

Foster RJ, Swiontkowski MF, Bach AW, et al: Radial nerve palsy caused by open humeral shaft fractures.  J Hand Surg Am 1993;18:121-124.

 

96.    The fracture shown in Figure 50 is most reliably treated with what form of fixation?

 

1-         Compression screws only

2-         Tension band wires and Kirschner wires

3-         Posterior plate

4-         Medial plate

5-         Flexible intramedullary rod

 

PREFERRED RESPONSE: 3

 

DISCUSSION: The radiograph shows a comminuted proximal ulnar fracture.  The most reliable fixation is a posterior plate, acting as a tension band plate.  The fracture involves the proximal shaft of the ulna; therefore, a 3.5-mm compression plate or one of similar size should be used to provide adequate stability.  Kirschner wires and tension band wires do not provide axial stability of the comminution of the ulna.  Compression screws alone will most likely fail and will not provide axial rotational stability to the construct.  A medial plate will not resist the distraction forces across this fracture. 

 

REFERENCES: McKee MD, Seiler JG, Jupiter JB: The application of the limited contact dynamic compression plate in the upper extremity: An analysis of 114 consecutive cases.  Injury 1995;26:661-666.

McKee MD, Jupiter JB: Trauma to the adult elbow and fractures of the distal humerus, in Browner BD, Jupiter JB, Levine AM, et al (eds): Skeletal Trauma, ed 2.  Philadelphia, PA, WB Saunders, 1998, p 1469.

 

97.    A man sustained the injury shown in Figures 51a and 51b.  He underwent closed reduction of the radial head dislocation and open reduction and internal fixation of the ulnar fracture.  What is the most common cause of persistent radial head subluxation?

 

1-         Interosseous ligament disruption

2-         Annular ligament disruption

3-         Avulsion of the common extensor origin

4-         Malreduction of the ulnar fracture

5-         Intra-articular osteochondral debris

 

PREFERRED RESPONSE: 4

 

DISCUSSION: The radiographs reveal a Monteggia injury, with a proximal ulnar shaft fracture and a radial head dislocation.  Treatment involves open reduction and internal fixation of the ulnar fracture.  With correct reduction of the ulna, the radial head is reducible and remains stable, despite an obvious soft-tissue injury around the elbow. Problems with persistent radial head subluxation are almost always attributed to malreduction of the ulnar fracture.  Rare causes of persistent radial head subluxation are interposition of soft tissues in the joint and lateral ligamentous injuries.

 

REFERENCES: Jupiter JB, Kellam JF: Diaphyseal fractures of the forearm, in Browner B, Jupiter J, Levine A, Trafton P (eds): Skeletal Trauma, ed 2.  Philadelphia, PA, WB Saunders, 1992, pp 1421-1454.

Ring D, Jupiter JB, Simpson NS: Monteggia fractures in adults.  J Bone Joint Surg Am 1998;80:1733-1744.

 

98.    Which of the following is an indication for surgical management of a Weber type B distal fibular fracture?

 

1-         Medial hindfoot ecchymosis

2-         Medial ankle tenderness

3-         Widened medial clear space

4-         Comminution

5-         Two millimeters of fibular displacement

 

PREFERRED RESPONSE: 3

 

DISCUSSION: A widened medial clear space indicates instability caused by an associated tear of the deltoid ligament; therefore, nonsurgical management is not warranted.  Weber type B distal fibular fractures without a deltoid tear have a medial clear space of < 4 mm, even under stress, and may be successfully treated nonsurgically.  The presence of medial hindfoot ecchymosis and medial ankle tenderness is not fully indicative of medial soft-tissue instability; however, these findings may indicate a deltoid injury and should raise suspicion of an unstable fracture injury pattern.  Stress testing is necessary to demonstrate the presence or absence of instability.  A small amount of comminution is also cause for increased suspicion of an unstable pattern; however, it is not a direct contraindication when considering nonsurgical management.  Two millimeters of fibular displacement without lateral shift of the talus is an acceptable position when considering nonsurgical management of Weber type B distal fibular fractures.

 

REFERENCES: Michelson JD, Magid D, Ney DR, et al, Examination of the pathologic anatomy of ankle fractures.  J Trauma 1992;32:65-70.

Marsh JL, Saltzman CL: Ankle fractures, in Rockwood & Green’s Fractures in Adults, ed 5.  Philadelphia, PA, Lippincott Williams and Wilkins, 2001, pp 2001-2090.

 

99.    Locked plating techniques have been shown to have biomechanical advantages over standard plating in which of the following scenarios?

 

1-         All osteoporotic fractures

2-         All comminuted fractures

3-         Spiral fractures

4-         Osteoporotic fractures with torsion

5-         Osteoporotic fractures without cortical contact

 

PREFERRED RESPONSE: 5

 

DISCUSSION: Locked plating is becoming more common.  Some biomechanical data comparing locked plating to standard plating have been reported for osteoporotic distal femoral fractures and humeral shaft fractures.  Significant differences were seen mainly for osteoporotic fractures without cortical contact.  Not all osteoporotic fractures and all comminuted fractures have been shown to demonstrate significant mechanical improvement with locked plating compared to standard plating.  Spiral fractures often can be repaired with a lag screw, obtaining adequate cortical contact.  Osteoporotic fractures with a torsion mode of failure failed earlier with locked plating systems than with standard plating systems. 

 

REFERENCES: Zlowodzki M, Williamson S, Cole PA, et al:  Biomechanical evaluation of the less invasive stabilization system, angled blade plate, and retrograde intramedullary nail for the internal fixation of distal femur fractures.  J Orthop Trauma 2004;18:494-502.

Comparison of the AO Locking Plate with the Standard Limited-Contact Dynamic Compression Plate (LC-DCP) for Fixation of Osteoporotic Humeral Shaft Fractures.  David J. Hak, MD, MBA; Scott J. Hazelwood, PhD.  OTA Book of Abstracts 2003.

 

100.  A 25-year-old man sustained a head injury after being ejected from his car.  Examination reveals a Glasgow Coma Scale score of 7 and a swollen right knee.  Clinical examination shows that the knee is very unstable, suggesting tears of the medial collateral and anterior and posterior cruciate ligaments, as well as the posterior lateral corner.  What is the most appropriate first step to rule out a vascular injury?

 

1-         Examination of the pedal pulses

2-         Ankle-brachial pressure index

3-         Duplex ultrasound

4-         Arteriography

5-         Exploration of the popliteal artery

 

PREFERRED RESPONSE: 1

 

DISCUSSION: A knee dislocation carries the potential for an arterial injury and has always brought up the question of need for arteriography to rule out this limb-threatening injury.  However, arteriography has an inherent complication rate that may compromise the general care of the patient.  In over 240 published cases with documented knee dislocations that were evaluated for vascular injury by physical examination (without imaging studies), not a single missed injury was reported, for a 100% negative predictive value (0% false-negative rate).  This degree of accuracy at excluding major vascular injury is unsurpassed by the results obtained with arteriography but with no risk involved and a marked savings in time, equipment, and costs.  Therefore, the most appropriate first step to rule out vascular injury is examination of the pedal pulses.  If there is any doubt about an arterial injury, another option is the ankle-brachial index (ABI).  If the ABI is greater than 0.9, the chance of arterial injury is again nonexistent.  However, a positive physical examination or an ABI of less than 0.9 is not 100% predictive of an arterial injury; therefore, arteriography is recommended.

 

REFERENCES: Miranda FE, Dennis JW, Veldenz HC, et al: Confirmation of the safety and accuracy of physical examination in the evaluation of knee dislocation for injury of the popliteal artery: A prospective study.  J Trauma 2002;52:247-252.

Mills WJ, Barei DP, McNair P: The value of the ankle-brachial index for diagnosing arterial injury afterknee dislocation: A prospective study.  J Trauma 2004;56:1261-1265.